Pre NEET Obstetrics and Gynaecology Sakshi Arora
INDEX
×
Chapter Notes

Save Clear


OBSTETRICS: New Clinical Question of Obstetrics

3
zoom view
4
5Pre NEET Obstetrics and Gynaecology
Questions
 
HEART DIS IN PREGNANCY
1. Which cardiovascular change is not physiological in preg­­nancy:
  1. Split 1st heart sound
  2. Middiastolic murmur
  3. Shift of apex beat to 4th ICS and outwards
  4. Decr peripheral vascular Resistance.
2. Indication for cesarean section in pregnancy is:
  1. Mitral stenosis
  2. Aorti c aneurysm
  3. PDA
  4. Transposition of great vessels
3. Surgery for mitral stenosis during pregnancy is done at:
  1. 8 wks
  2. 10 wks
  3. 14 wks
  4. 22 wks
4. Which of the following disease has worst prognosis during pregnancy:
  1. Pulmonary stenosis
  2. Mitral stenosis
  3. VSD
  4. ASD
65. All of the following are predictors of cardiac event during pregnancy except:
  1. NYHA class>3
  2. Obstructive lesion of the heart (mitral valve and aortic valve <1 cm2
  3. Previous H/O heart failure
  4. Ejection fraction <40%.
Case Study
  • P2L2 patient, on the 3rd postoperative day of caesarean develops sudden cardiac failure.
    • – She has weakness, shortness of breath, palpitation, nocturnal dyspnea and cough.
    • – O/E- Tachycardia, arrhythmia, peripheral edema, pulmonary rales are present. S3 is present but no murmur is heard.
    • – She had been a booked patient with regular antenatal check-ups and with no prior heart problem and uneventful prior obstetric history.
Q. What is the probable diagnosis?
 
DIABETES IN PREGNANCY
6. A 30-year-old woman with diabetes mellitus presents to her physician at 19 weeks’ gestation. She is obese and did not realize that she was pregnant until recently. She also has not been “watching her sugar” lately, but is now motivated to improve her regimen. A dilated ophthalmologic examination shows no retinopathy. An ECG is normal. Urinalysis is negative for proteinuria. Laboratory studies show:
  • Hemoglobin A 1c: 10.8%
  • Glucose: 222 mg/dL
  • Thyroid-stimulating hormone: 1.0 µU/mL
  • Free thyroxine: 1.7 ng/dL
  • Creatinine: 1.1 mg/dL.
Q. In which of the following condition the risk of developing it is same in diabetics as the general population.
  1. Asymptomatic bacteriuria
    7
  2. Preeclampsia
  3. Congenital adrenal hyperplasia
  4. PPH after delivery
  5. Shoulder dystocia
7. 30-yrs-old G3P2 patient visits an antenatal clinic at 20 weeks. She reveals during history that her first baby was 4.6 kg delivered by cesarean section, second baby was 4-8 kg delivered by c/section. Gynaecologists suspects gestational diabetes and orders a GCT. The blood sugar levels after 50 gms of oral glucose are 206 mg/dl and the patient is thus confirmed as a case of gestational diabetes. All of the following are known complications of this condition except:
  1. Susceptibility for infection
  2. Fetal hyperglycemia
  3. Congenital malformations in fetus
  4. Neonatal hypoglycemia
8. A 30-year-old G3P2 obese woman at 26 weeks’ gestation with no significant past medical history states that diabetes runs in her family. Her other pregnancies were uncomplicated. The results of a 3-hour glucose tolerance test show the following glucose levels:
  • 0 (fasting): 90 mg/dL 1 hour: 195 mg/dL
  • 2 hours: 155 mg/dL 3 hours: 145 mg/dL
    As a result, she is diagnosed with gestational diabetes. She is counselled to start diet modification and exercise to control her glycemic levels. 3 weeks after her diagnosis, she presents her values:
  • Fasting: 95 mg/dL 1hr pp: 185 mg/dL
Q. What is the best management:
  1. Continue diet modification
  2. Start insulin
  3. Repeat GTT
  4. Start metformin
9. Fasting Blood sugar should be mantained in a pregnant diabetic female as:
  1. 70 – 100 mg%
  2. 100 – 130 mg%
  3. 130 – 160 mg%
  4. 160 – 190 mg%
8
 
ANEMIA IN PREGNANCY
10. Total amount of iron needed by the fetus during entire pregnancy is:
  1. 500 mg
  2. 1000 mg
  3. 800 mg
  4. 300 mg.
11. Thirty years old G4P3L3 with 32 weeks pregancy with single live fetus in cephalic presentation, Patient complains of easy fatiguability and weakness since last 3 months which has gradually increased over last 15 days to an extent that she gets tired on doing household activities. Patient also complaints of breathlessness on exertion since last 15 days. Patient gets breathless on climbing 2 flight of stairs. It is not associated with palpitations or any chest pain. There is no history of pedal edema, sudden onset breathlessness, cough or decreased urine output. There is no history of asthma or chronic cough. There is no history of chronic fever with chills or rigors. There is no history of passage of worms in stool nor blood loss from any site. There is no history of easy bruisability or petechiae. There is no history of yellow discoloration of urine, skin or eyes. She did not take iron folate prophylaxis throughout her pregnancy.
  • She is suspected to be anemic and her blood sample was ordered for examination which showed.
  • Hb 7.4 gm % (12–14 gm%)
  • Hct 22 % (36–44%)
  • MCV 72 fL (80–97 fL)
  • MCH 25 pg (27–33 pg)
  • MCHC 30 % (32–36%)
  • Peripheral smear shows microcytic hypochromic RBCs with anisopoikilocytosis
  • Naked eye single tube red cell osmotic fragility test (NESTROFT) is negative.
Q. What is the most probable diagnosis:
  1. Thallesemia
  2. Iron deficiency anemia
  3. Megaloblastic anemia
  4. Vit B12 deficiency anemia.
9
 
INFECTIONS IN PREGNANCY
12. A 6 week pregnant lady diagnosed with sputum positive TB. Best management is:
  1. Wait for 2nd trimester to start ATT
  2. Start category 1 ATT in 1st Trimester
  3. Start category 1 1 ATT in 1st Trimester
  4. Start category 111 ATT in 2nd Trimester
13. A 32-year-old G2P1 woman at 34 weeks’ gestation presents to the labor and delivery floor with the chief complaint of regular contractions, bloody show, and a gush of fluids. A 2.3 kg (5 lb 1 oz) boy is delivered by spontaneous vaginal delivery without further complication 1 hour after presentation. Twenty-four hours later, the infant has developed irritability, fever, and respiratory distress. He is diagnosed with sepsis secondary to pneumonia. The mother has no complaints other than anxiety regarding the condition of her child. She denies rigors, chills, sweats, nausea, or vomiting. The mother's pulse is 60/min, blood pressure is 125/80 mm Hg, and temperature is 37°C (98.6°F). Physical examination reveals lungs that are clear to auscultation bilaterally, and no murmurs, rubs, or gallops are present on cardiac examination. The suprapubic region is not tender to palpation. Vaginal and cervical examination reveals no significant tears or bleeds.
Q. Which prenatal test would have provided the most useful information in preventing this condition:
  1. Cervical Chlamydia culture
  2. Cervical gonorrhea culture
  3. Elisa for HIV
  4. Rectovaginal grp B streptococcal culture
14. A 37-year-old G2P1 woman at 38 weeks’ gestation presents to the obstetrics clinic for a prenatal visit. The patient had difficulty becoming pregnant but was successful after using in vitro fertilization. She has a history of recurrent herpes outbreaks, and her first pregnancy was complicated by failure to progress, which resulted in a cesarean birth. Routine rectovaginal culture at 36 weeks was positive for Group B streptococci.
Q. Which of the following would be an absolute indication for delivering the child by LSCS:
10
  1. Current symptoms of genital pain and tingling
  2. h/o previous cesarean section
  3. IVF
  4. Maternal colonization with group B streptococci
15. A 25-year-old G1P0 female at 25 wks of gestation comes to you for antenatal check up. She has had an uncomplicated pregnancy but has 5 years history of Genital Herpes infection. She is usually asymptomatic and has had 3 flares in the past 5 years. She is concerned about exposing her unborn child to infection-What is the most appropriate counsel to offer to this patient.
  1. Administer one dose of acyclovir if she has active genital herpes at the time of delivery.
  2. Administer prophylaxis with acyclovir from now and uptil delivery whether she has active herpes or not.
  3. Perform elective LSCS even if mother is asymptomatic at the time of delivery.
  4. Perform elective LSCS only if mother has active herpes at the time of delivery.
16. A 26-Year-old woman is 38 weeks pregnant and presents to the labour room in active labour. She had fever for past 2days. Last night she broke out in any itchy rash that has spread over her arms and torso. She is a teacher by profession and 2 weeks earlier one of the children in her class was diagnosed with chicken pox. She didn't have chickenpox as a child.
Q. The patient is worried: Which of the following is the best advice to give her:
  1. Nothing needs to be done, chickenpox in children is mild and self limiting.
  2. The chance of transmitting the virus of the baby is low and so we treat if symptoms develop.
  3. Baby must be treated immediately after birth as chickenpox is serious in newborns
  4. Varicella virus is teratogenic and baby might have mild birth defects.
17. A 34-year-old primigravida at 11 weeks gestation presents to her obstetrics clinic with chief complain of exposure to a rash. Her husband is HIV+ve and has broken out on a rash in his left buttock which 11consists of a grouped vesicles on a maculopapular base, 4 days back. She has got her HIV testing done which is negative. Her P/R is 86/min, B/P = 100/60 mm of hg, resp rate 10/min and temp = 98.7F.FHS is heard via Doppler.
Q. What is the next step in the management:
  1. Administer high dose acyclovir to the infant at birth.
  2. Administer high dose acyclovir to the patient now.
  3. Administer varicella immunoglobulin to the infant at birth.
  4. Administer varicella immunoglobulin to the patient.
18. A 34 year old multigravida at 32 weeks gestation pre­sents to her obstetrics clinic with reports positive for Hbsag. Which of the following statements concerning hepatitis infection in pregnancy are true:
  1. Hepatitis B core antigen status is the most sensitive indicator of positive vertical transmission of disease
  2. Hepatitis B is the most common form of hepatitis after blood transfusion
  3. The proper treatment of infants born to infected mothers includes the administration of hepatitis B Ig as well as vaccine.
  4. Patients who develop chronic active hepatitis should undergo MTP.
19. In an HBsAg+ve Pregnant female, hepatitis Ig to the child should be given: (AIPG 2012)
  1. Within 12 hours
  2. Within 6 hrs
  3. Within 24 hrs
  4. Within 48 hrs
20. A 19-year-old G2P1 woman at 9 weeks’ gestation presents to the obstetrics and gynaecology clinic for her second prenatal visit. She reports no complaints other than occasional nausea. She had her first child by spontaneous vaginal delivery without complications. She is taking no medications and denies ethanol, tobacco, or current drug use. While she does admit to a history of intravenous drug abuse, she denies using them since the birth of her first child. Over the past several months she has had multiple sexual partners and does not use contraception. On physical examination she is in no acute distress. Lungs are clear to auscultation bilaterally. Her heart has a regular rate and rhythm, with no murmurs, rubs, or gallops. She is informed that she will need the 12routine prenatal tests, including an HIV test. The physician informs her of the risks and benefits of the HIV test.
Q. What else should the physician inform the patient before performing the test:
  1. Despite the potential for fetal infection, she may opt out from the test
  2. Early retroviral therapy will absolutely decrease the chances of transmitting infection to the baby.
  3. CDC recommends screening only for patients with high risk factors
  4. Risk of the test include potential for fetal loss.
 
CASE STUDY
A primigravida female of 32 years is 8 weeks pregnant and is a diagnosed case of HIV.She is already on ART and has no problem otherwise. What is the recommended treatment for her.
21. A primigravida delivers a premature infant (35 weeks) with bullous lesions all over the skin. X ray evaluation of bones of extremities shows periostitis. Which of the following investigation is useful in making the diagnosis:
  1. VDRL in mother and baby
  2. HbS ag
  3. Montoux test
  4. ELISA for HIV
22. DOC for syphilis in pregnancy = AIPG2012:
  1. Erythromycin
  2. Azithromycin
  3. Penicillin
  4. Cephalosporin/ceftriaxone
 
FIRST TRIMESTER BLEEDING
23. An 18-year-old woman complains of lower abdominal pain and vaginal spotting for several days. She denies sexually transmitted disease although she is sexually active with her boyfriend; they use condoms for protection. Her last menstrual period was 6 weeks ago. Her blood pressure is 124/80 mm Hg, pulse is 90/min, and temperature is 37.2°C (99.0°F). Abdominal examination demonstrates vague left lower quadrant 13tenderness without rebound or guarding. Pelvic examination shows a normal vagina and cervix without cervical motion tenderness. No adnexal masses are appreciated. Results of a complete blood cell count and metabolic panel are within normal limits.
Q. Which of the following is the next best step in mgt:
  1. Transvaginal usg
  2. Follow up after 3 months
  3. Quantitative b hcg measurement
  4. Rapid urine b hcg measurement
  5. Methotrexate inj.
24. A 29-year-old G1P1 woman presents to the clinic for a prenatal check-up at 10 weeks’ gestation with concerns of brown vaginal discharge about 1 week ago. She has noticed that her stomach is no longer increasing in size and that she is no longer as nauseated. On physical examination the cervix is closed and the uterus is impalpable. Ultrasound reveals a normal appearing 6 week fetus, but no fetal heartbeat.
Q. Which of the following is the most likely diagnosis:
  1. Incomplete abortion
  2. Missed abortion
  3. Threatened abortion
  4. Complete abortion
  5. Inevitable abortion.
25. A woman with H/O recurrent abortions presents with isolated increase in APTT. Most likely cause is:
  1. Lupus anticoagulant
  2. Factor vii
  3. Von willebrands disease
  4. Hemophilia.
26. A Patient at 22 weeks gestation is diagnosed as having IUD which occurred at 17 weeks but did not have a miscarriage. This patient is at increased risk for:
  1. Septic abortion
  2. Recurrent abortion
  3. Consumptive coagulopathy with hypofibrinogenemia
  4. Future infertility
  5. Ectopic pregnancy
1427. A 36-year-old G1P0 woman presents for her first prenatal visit late in her first trimester of pregnancy; she complains of persistent vaginal bleeding, nausea, and pelvic pain. Physical examination is notable for a gravid uterus larger than expected for gestational age. Fetal heart tones are absent. The patient has a medical history significant for herpes and gonorrhea infections.
Q. Which of the following is most likely to be true:
  1. B hcg levels will be higher than normal.
  2. B hcg levels will be lower than normal
  3. uterus will be of normal levels
  4. TSH levels will be increased
 
APH
28. A 34-year-old G1P0 woman at 29 weeks’ gestation presents to the emergency department complaining of 2 hours of vaginal bleeding. The bleeding recently stopped, but she was diagnosed earlier with placenta previa by ultrasound. She denies any abdominal pain, cramping, or contractions associated with the bleeding. Her temperature is 36.8°C (98.2°F), blood pressure is 118/72 mm Hg, pulse is 75/min, and respiratory rate is 13/min. She reports she is Rh-positive, her hemoglobin is 11.1 g/dL, and coagulation tests, fibrinogen, and D-dimer levels are all normal. On examination her gravid abdomen is nontender. Fetal heart monitoring is reassuring, with a heart rate of 155/min, variable accelerations, and no decelerations. Two large-bore peripheral intravenous lines are inserted and two units of blood are typed and crossed.
Q. What is the most appropriate next step in management:
  1. Admit to antenatal unit for bed rest and betamethasone.
  2. Admit to antenatal unit for bed rest and blood transfusion.
  3. Induction of labour
  4. Perform emergency cesarean section.
29. A 29 year old G3 P2 female at 32 weeks of gestation presents to the emergency dept. with a small amount of vaginal bleeding. She doesn't have any pain.
  • On examination
  • Her PR : 66/min
    15
  • B/P : 100/70mm of hg
  • RR : 10/min
FHS tracings show fetal distress and shows late decelerations.
Q. What is the best course of action:
  1. Emergent cesarean section
  2. Fetal umbilical blood transfusion
  3. Expectant management
  4. Induction of labour with prost aglandins
30. A 29-year-old G3P2 woman at 34 weeks’ gestation is involved in a serious car accident in which she lost consciousness briefly. In the emergency department she is awake and alert and complains of a severe headache and intense abdominal and pelvic pain. Her blood pressure is 150/90 mm Hg, heart rate is 120/min, temperature is 37.4°C (99.3°F), and respiratory rate is 22/min. Fetal heart rate is 155/min. Physical examination reveals several minor bruises on her abdomen and limbs, and vaginal inspection reveals blood in the vault. Strong, frequent uterine contractions are palpable.
Q. Which of the following is most likely a complication of this pts present condition:
  1. DIC
  2. IUGR
  3. Subarachnoid hemorrhage
  4. Vasa previa
31. A 34-year-old G1P0 woman at 30 weeks’ gestation with a medical history of hypertension and tobacco use presents to the emergency department because of 3 hours of spontaneous vaginal bleeding. She is lethargic and complains of severe abdominal pain. Her temperature is 37.1°C (98.8°F), blood pressure is 82/44 mm Hg, pulse is 125/min, and respiratory rate is 18/min. Abnormal results of laboratory tests show an International Normalized Ratio of 2.3 and a partial thromboplastin time of 48 seconds. D-dimer levels are elevated, and fibrinogen levels are decreased. Fetal heart monitoring is concerning for an absent fetal heart rate.
Q. Which of the following is the m ost likely cause of this patients abnormal lab tests:
  1. Disruption of placenta and release of fetal tissue into circulation
    16
  2. Liver failure
  3. Haemorrhagic shock
  4. Release of thromboplastin by damaged placenta
  5. Ruptured ectopic
32. A 27-year-old G2P1 woman at 34 weeks’ gestation presents to the emergency department following a motor vehicle collision. In the trauma bay her heart rate is 130/min and blood pressure is 150/90 mm Hg. She is alert and oriented to person, place, and time. She complains of severe abdominal pain that began immediately after the collision. Physical examination reveals bruising over her abdomen, along with a hypertonic uterus and dark vaginal bleeding. A sonogram reveals a placental abruption, and the fetal heart tracing reveals some decelerations. Emergency laboratory tests reveal an International Normalized Ratio of 2.5, with elevated fibrin degradation products.
Q. Which of the following is the most appropriate first step in management:
  1. Administer a tocolytic
  2. Administer a corticosteroid.
  3. Administer fresh frozen plasma.
  4. Deliver the fetus immediately by LSCS
  5. Observe closely.
 
PIH
33. A 31-year-old G2P1 woman at 24 weeks’ gestation presents for a routine prenatal visit. She reports an uneventful pregnancy other than early morning nausea and vomiting, which has subsided since her last visit. She denies vaginal bleeding or contractions. Blood pressure and routine laboratory values at previous visits had been normal. Today her temperature is 37°C (98.6°F), pulse is 74/min, blood pressure is 162/114 mm Hg, and respiratory rate is 14/min. Her uterine size is consistent with her dates, and her physical examination is unremarkable. Laboratory tests show:
  • WBC count: 9000/mm³
  • Hemoglobin: 13 g/mL
  • Hematocrit: 39%
  • Platelet count: 240,000/mm³
    17
  • Blood urea nitrogen: 11 mg/Dl
  • Creatinine: 1.0 mg/dL
  • Aspartate aminotransferase: 20 U/L Alanine aminotransferase: 12 U/L.
  • Urinalysis reveals 3+ protein but no blood, bilirubin, bacteria, leukocyte esterase, or nitrites. The patient is sent directly from the clinic for a nonstress test and an ultrasound. Six hours later her blood pressure is rechecked, and it is 162/110 mm Hg.
Q. Which of the following is the most likely diagnosis-
  1. Chronic hypertension
  2. Preeclampsia
  3. Eclampsia
  4. Gestational hypertension
  5. Severe preeclampsia
34. A 32-year-old G3P2 woman at 35 weeks’ gestation has a past medical history significant for hyper­tension. She was well-controlled on hydrochlorothiazide and lisinopril as an outpatient, but these drugs were discontinued when she found out that she was pregnant. Her blood pressure has been relatively well controlled in the 120-130 mm Hg systolic range without medication, and urinalysis has consistently been negative for proteinuria at each of her prenatal visits. She presents now to the obstetric clinic with a blood pressure of 142/84 mm Hg. A 24-hour urine specimen yields 0.35 g of proteinuria.
Q. Which of the following is the most appropriate next step?
  1. Start iv furosemide
  2. Induce labor after doing Bischop score
  3. Put her on hydralazine
  4. Initial inpatient evaluation followed by restricted activity and outpatient management.
  5. Start her prepregnancy regime
35. A 35 years old G1 P0 women at 28 weeks of pregnancy complaints of severe headache for 4days. She doesn't have any photophobia, vomiting and nausea but had dizzness. Her BP is 155/85mm of hg, R/R-18/min, P/R-120/min.
Urinalysis reveals +1 glycosuria, +3 proteinuria and 24 hours urine collection shows 4g protein.
18P/A Examination: ht of uterus ~ 28 weeks:
  • FHS regular
  • Fetal parts palpable
She is admitted and monitored after 6hrs her condition is unchanged which of the following is the next best step in management:
  1. Emergency cesarean section
  2. Oral glucose tolerance test
  3. I/V mgso4
  4. Stabilisation of vital signs and bed rest
  5. Follow up after 2 weeks
36. A 25 Year old femal is 5months pregnant and presents to her obstetrician along with her first child. She has not received any prenatal care. She thinks she has gained adequate weight and her pregnancy has been uncomplicated till date. Her past medical history is notable for hypertension for which she is currently taking enalapril.
  • She is 168 cms (5’ 6”) tall, weight is 59 kg, B/P = 120/84 mm of hg and fundal ht is 17 cms. Fetal movements are appreciated and FHR = 140/min.
  • Results of dipstick are negative.
  • Which of the following tests should be preformed:
  1. CVS
  2. Grp B strepto coccal testing
  3. Triple test
  4. USG of fetal kidneys
 
AMNIOTIC FLUID DISORDERS
37. A multigravida 32 years old female presents at 30 weeks of pregnancy for routine examination. She has history of type-2 Diabetes Mellitus, Hypercholesteremia and hypertension and has a 5back years smoking history. She is Rh positive and husbands Rh status is unknown.
  • USG shows AF I>21
  • Modification of which of the following would most likely have helped to prevent this condition:
  1. Folate supplementation
  2. Hypertension
    19
  3. Rh Isoimmunisation
  4. Diabetes
  5. Smoking
 
RH NEGATIVE PREGNANCY
38. A 37 year old primi Rh negative patient is very concerned abt her pregnancy at this age. Her pregnancy is 16 weeks and she is HIV negative, hepatitis b surface ag neg, Rubella non immune and has no complain. Her triple test report is normal but still due to her age she insists on getting an amniocentesis done.
Which of the following is the next best step in management:
  1. Advise against amniocentesis as it will increase the risk of isoimmunisation
  2. Follow Rh titres carefully and give Anti D if evidence of isoimmunisation is present.
  3. Give Anti D at 28 weeks f pregnancy and after delivery if baby is Rh neg
  4. Give Anti D prior to her amniocentesis
  5. give rubella vaccine as she is Rubella non immune
39. Two weeks later, the results of the patient's prenatal labs come back. Her blood type is A, with an anti D antibody titer of 1:4. What is the most appropriate next step in the management of this patient?
  1. Schedule an amniocentesis for amniotic fluid bilirubin at 16 weeks
  2. Repeat titer in 4 weeks
  3. Repeat titer in 28 weeks
  4. Schedule PUBS to determine fetal hematocrit at 20 weeks
  5. Schedule PUBS as soon as possible to determine fetal blood type
40. All of the following are scenarios in which it would have been appropriate to administer RhoGam to this patient in the past except:
  1. After a spontaneous first trimester abortion
  2. After treatment for ectopic pregnancy
  3. Within 3 days of delivering an Rh–ve fetus
  4. At the time of amniocentesis
  5. At the time of external cephalic version
20
 
JAUNDICE IN PREGNANCY
41. A pregnant female developed idiopathic cholestatic jaundice. The following condition is not associated:
  1. Intense itching
  2. SGOT, SGPT less than 60 IU
  3. Serum bilirubin > 5 mg/dl
  4. Markedly raised levels of alkaline phosphatase
42. Pregnancy shd be terminated at …..wks in pts of cholestatic jaundice:
  1. 34 weeks
  2. 36 weeks
  3. 38 weks
  4. 40 weeks
43. A 36 yr old G1P0 at 35 weeks gest presents with several days H/O generalised malaise, anorexia, nausea and emesis and abd. discomfort. She has loss of apetite and loss of several pounds wt in 1 week. Fetal movements are good. There is no headache, visual changes, no vaginal bleeding, no regular uterine contractions or rupture of membranes. She is on prenatal vitamins. No other medical problem. On exam she is mild jaundiced and little confused. Her temp is 100 degree F, PR- 70, BP- 100/62, no significant edema, appears dehydrated. FHR is 160 and is nonreactive but with good variability. Her WBC- 25000, Hct- 42.0, platelets- 51000, SGOT/SGPT- 287/ 350, GLUCOSE-43, Creatinine- 2.0, fibrinogen- 135, PT/PTT- 16/50, S. Ammonia level- 90 micromol/L. Urine is 3+ Proteins with large amount of ketones.
What is the recommened treatment for this patient.
  1. Immediate delivery
  2. Cholecystectomy
  3. Intravenous diphenhydramine
  4. MgSO4 therapy
  5. Bed rest and supportive measures since this condition is self limiting
 
ASTHMA IN PREGNANCY
44. A 23-year-old G1P0 woman at 10 weeks’ gestation presents to the obstetrics clinic for her initial evaluation. She says she has been 21hospitalized several times for asthma exacerbations but has never required intubation or admission to an intensive care unit. She is controlled on daily inhaled corticosteroids and albuterol with adequate relief of her symptoms. She is concerned about taking these medications now that she is pregnant.
Q. Which of the following is true regarding asthma medications in pregnancy:
  1. B2 agonist are contraindicated during pregnancy.
  2. Both B2 agonist and inhaled corticosteroids are both contraindicated in pregnancy.
  3. Both B2 agonist and inhaled corticosteroids are safe in pregnancy
  4. B2 agonist and inhaled corticosteroids are both safe in pregnancy but during 2nd and 3rd trimester only
  5. Inhaled corticosteroids are contraindicated in pregnancy.
 
THYROID DISORDERS IN PREGNANCY
45. Rani a 24 year old woman presents to her gynae­cologist as she has chronic hypothyroidism and wants to conceive now. Her hypothyroidism is well controlled at 75 microgram of Thyroxine. She doesn't smoke or drink and doesn't have any other medical ailment. She would like to know if she should keep taking her Thyroxin. Which of the following is the best advice to give to this patient:
  1. Stop taking Thyroxine and switch to methimazole as we would like to control your baby's thyroid levels
  2. Thyroxine is safe during pregnancy but it is not absolutely necessary during pregnancy to continue thryo­xine.
  3. Thyroxine is not safe during pregnancy and it is better for your baby to be hypothyroid than hyperthyroid
  4. Thyroxine is absolutely safe and necessary for you in pregnancy but we would like to decrease your dose as pregnancy is accompanied by mild physiological hyperthyroidism
  5. Thyroxine is safe in pregnancy and the dose of thyroxine would be increased during pregnancy to avoid hypothyroidism, which may affect the baby adversely.
22
 
TWIN/MULTIFETAL PREGNANCY
46. Which of the following statements about twinning is true?
  1. The frequencies of monozygosity and dizygosity are the same
  2. Division after formation of the embryonic disk result in conjoined twins
  3. The incidence of monozygotic twinning varies with race
  4. A dichorionic twin pregnancy always denotes dizygosity
  5. Twinning causes no appreciable increase in maternal morbidity and mortality over singleton pregnancies
47. The placenta of twins can be:
  1. Dichorionic and monoamniotic in dizygotic twins
  2. Dichorionic and mooamniotic in monozygotic twins
  3. Monochorionic and monoamniotic in dizygotic twins
  4. Dichorionic and diamniotic in monozygotic twins
48. A 26 yr old primigravida with a twin gestation at 30 weeks presents for a USG.The sonogram indicates that the fetuses are both male and the placenta appears to be diamniotic and monochorionic.Twin B is noted to have oligohydramnios and to be much smaller than twin A.In this clinical scenario, all of the following are concerns for twin A except:
  1. CHF
  2. Anemia
  3. Hydramnios
  4. widespread thromboses
 
LABOUR/ABNORMAL LABOR
49. All of the following are appropriate tocolytics for this patient except:
  1. indomethacin
  2. Nifedipine
  3. MgSO4
  4. Progesterone
50. Best tocolytic in a cardiac patient is:
  1. Atosbian
  2. Isoxsuprine
  3. Nifedipine
  4. Mgso4
2351. Which of the following clinical conditions is not an indication for induction of labour:
  1. IUD
  2. Severe preeclampsia at 36 weeks
  3. complete placenta accreata
  4. Chorioamnionitis
  5. Postterm pregnancy
52. Active management of third stage of labour includes all except:
  1. Uterine massage
  2. Delivery of placenta by controlled cord traction
  3. Early cord clamping
  4. Injection methergin after delivery of shoulder of baby
53. All of the following are indications for early clamping of cord except:
  1. Preterm delivery
  2. Postdated pregnancy
  3. Birth asphyxia
  4. Maternal diabetes
54. A 27-year-old G1P0 woman at 39 weeks’ gestation presents to the labor and delivery suite and progresses through the stages of labor normally. During delivery of the infant, the head initially progresses beyond the perineum and then retracts. Gentle traction does not facilitate delivery of the infant.
Q. Which of these options is the first step in the management:
  1. Abduct mothers thigh and apply suprapubic pressure.
  2. Apply fundal pressure.
  3. Flex mothers thigh against her abdomen.
  4. Push infants head back into the uterus and do cesarean section.
  5. Do a symphiosotomy.
55. A 28 year old G1P0 woman at 31 weeks gestation presents with 4hours history of abdominal cramping and contraction. She is feeling contractions at regular intervals of 10 mins and are increasing is intensity. She has had a small amount of vaginal discharge but is unsure whether her water bag has broken or not.
She has had no vagina bleeding
24Her temp is 36.80 C (98.30F), BP-137/84mm of Hg, pulse 87/min and R/R=12/min
On examination:
  • Uterus is soft
  • Fundal ht ≃ 32weeks
  • Contractions present 20-30secs/8mins
  • Cephalic presentation
  • FHS + Regular
Which of the following is the next best step in management:
  1. Cervical culture for group B streptococci
  2. Digital cervical examination and assessment of dilation and effacement.
  3. Quantification of strength and timing of contraction with external tocometer.
  4. Speculum examination to rule out leaking and usually assess cervical dilatation and effacement.
  5. USG examination of the fetus.
56. A primigravida at 37th weeks of gestation with loss of engagement, 1 cm effacement of cervix and 10 uterne contractions per hour. She is hemodynamically stable and not in distress. What is the management.  (AI 2011)
  1. Sedate the patient and wait
  2. LSCS
  3. Amniotomy
  4. Induction with membrane rupture
57. A 30 year old primigravida at 39 weeks has been completely dilated and pushing for 3 hrs. She has taken epidural analgesia.She is exhausted and her temp is 37.8^c. FHS is 170/min with decreased variability. Patients membranes are ruptured for over 24 hrs. P/V shows cervix is fully dilated and fetal head is visible in between contractions and fetal bones are at +3 station.What is the most appropriate management:
  1. do LSCS
  2. encourage the patient to push after a short period of rest.
  3. attempt forceps delivery
  4. apply fundal pressure
2558. A 32 year old G2P1 at 38 weeks of gestation presents to the labour room with regular intense painful uterine contractions for the past one hour.She belives her water bag has broken and has H/O previous LSCS for fetal distress.
  • O/E – P/R = 95/min
  • B/P = 135/8 o mmof hg
  • R/R – 15/min
  • P/A – fetus lie-long, presentation cephalic
  • Tocometer detcts contractions every 8 mins
  • Fetal heart rate tracing show baseline FHS 140/min, beat to beat variability is present, occasional heart rate acceleration of 160/min for 15–20 secs. There are also decelarations to 115–120/min with the onset of contractions.
What is the most appropriate next step in management?
  1. Augment contractions wid oxytocin
  2. Monitor and follow labor on partogram
  3. Obtain immediate consent for LSCS
  4. Send the patient for BPS
  5. Perform urgent aminoinfusion
59. A healthy 30 yr old G1P0 at 41 weeks presents to labor and delivery at 11 pm because the baby's movements were less for the past 24 hrs. the pregnancy period was without any complication. Her baseline BP was normal. FHR is 180 bpm with absent variabi­lity. Uterine contractions are every 3 min accompanied by FHR deceleration. Physical exam indicates cervix is long/ closed/-2. What is the appropriate plan of mgmt.
  1. Emergency CS
  2. IV MgSO4 and induce labor with with pitocin
  3. Overnight cervix ripening with PGE2 and induction with Pitocin in morning
  4. Admission and CS after 12 hrs of NPO
  5. Induce labor with misoprostil
60. A healthy 23 yr old G1P0 has an uncomplicated pregnancy to date. She is dissapointed because she is 41 weeks gestational age by good dates and a 1st trimester USG and wants to have her baby. Pt reports good fetal movements, baby's kick count is abt. 8–10 times/hr. 26On exam cervix is firm, posterior, 50% effaced and 1 cm dilated and vertex is at -1 stn. What will be the next advice for the pt.
  1. Admission and immidiate CS
  2. Admission and Pitocin induction
  3. Schedule a CS in one week if she has not undergone spontaneous labor in the mean time
  4. She should continue to monitor kick count and return to you after a week to reassess the situation
61. A 24 yr old primi female at term, has been dilated to 9 cms for 3 hrs.The fetal vertex is at Rt occipito posterior position and at +1 station.There have been mild decelerations for the last 10 mins.Twenty mins back fetal scalp Ph was 7.27 and now it is 7.20. Next line of management is:
  1. wait and watch
  2. repeat scalp ph after 15 mins
  3. midforceps rotation
  4. LSCS
62. A 27-year-old G2P1 woman at 40 weeks’ gestation presents in labor. She has a history of an uncomplicated spontaneous vaginal delivery of a healthy child weighing 3.9 kg (8.6 lb). On examination her blood pressure is 123/89 mm Hg, pulse is 87/min, and temperature is 36.7°C (98°F). The fetal heart rate ranges from 140 to 150/min with good beat-to-beat variability. Tocometry detects regular contractions occurring every 8-10 minutes. The cervix is dilated at 4 cm and the vertex is at the -3 position. Immediately after artificial rupture of membranes, fetal bradycardia of 65-75/min is noted for 2 minutes without recovery.
Which of the following is the next best step in mgt:
  1. incr rate of oxytocin infusion
  2. Perform sterile vaginal examination
  3. perform immediate LSCS
  4. Perform mc roberts manouvre
  5. stimulate fetal scalp
63. Treatment of cord prolapse is based on all of the following factors except:
  1. Fetal viabilty
    27
  2. Fetal maturity
  3. Fetal weight
  4. Cervical dilatation
64. A 37-year-old G2P1 woman at 38 weeks’ gestation, with regular prenatal care, presents to the labor and delivery floor after several hours of increasingly frequent and strong contractions with ruptured amniotic membranes. On examination her cervix is soft, anterior, and completely effaced and dilated. Labor continues for another 3 hours, at which time the fetus has still not been delivered. The fetal mean heart rate is 146/min, with variable accelerations and no appreciable decelerations. Evaluation of the fetus and maternal pelvis indicate that anatomic factors are adequate for vaginal delivery.
Which of the following is an indication for forceps delivery:
  1. Fetal distress during active stage of labor.
  2. Labor complicated by shoulder dystocia.
  3. Prolonged active stage of labor due to inadequate uterine contraction strength.
  4. Prolonged latent stage of labor due to inadequate uterine contraction strength.
  5. Prolonged second stage of labor due to adequate uterine contraction strength.
65. In the criteria for outlet forceps application all are correct except:
  1. Fetus should be vertex prestation or face presentation with mentoanterior
  2. The saggital plane should be less than 15 degree from anterioposterior plane
  3. There should be no caput saccedenum
  4. It should be at station zero (AIIMS Nov 2011)
66. Long axis of the forceps lie along which fetal diameter:
  1. mentovertical
  2. suboccipitobregmatic
  3. occipitofrontal
  4. occipitomental
2867. A forceps rotation of 30o from left occipito anterior with extraction of fetus from +2 station is described as which type of forceps application:
  1. High forceps
  2. Mid cavity forceps
  3. Low forceps
  4. Outlet forceps
68. An abnormal attitude is illustrated by:
  1. Breech presentation
  2. Face presentation
  3. Transverse position
  4. Occiput posterior
  5. Occiput anterior
69. A 30 yr old G1P1001 patient comes to see you in office at 37 weeks gestational age for her routine OB visit. Her 1st pregnancy resulted in a vaginal delivery of a 9-lb, 8-oz baby boy after 30 min of pushing. On doing Leopold maneuvers during this office visit, you determine that the fetus is breech. Vaginal exam demonstrate that the cervix is 50% effaced and 1–2 cm dilated. The presenting breech is high out of pelvis. The estimated fetal wt. is about 7 lb. you send the pt. for a USG, which confirms a fetus with a frank breech prestation. There is a normal amount of amniotic fluid present, and the head is well flexed. As the patient's obstetrician, you offer all the following possible mgmt plans except:
  1. Allow the pt to undergo a vaginal breech delivery whenever she goes into labor
  2. Send the pt to labor and delivery immidiately for an emergent CS
  3. Schedule a CS at or after 39 weeks gestation age
  4. Schedule an ext cephalic version in next few days
 
PUERPERIUM
70. Kegels exercise shd begin: (AIPG2012)
Version 1:
  1. immediately after delivery
  2. 24 hrs after delivery
  3. 3 weeks after delivery
  4. 6 weeks after delivery
29Version 2:
  1. Immediately after delivery
  2. 3 weeks after delivery
  3. Only after LSCS
  4. During third trimester of pregnancy
71. A 24-year-old P2+0 woman presents to the emergency department complaining of pain in her right breast. The patient is postpartum day 10 from an uncomplicated spontaneous vaginal delivery at 42 weeks. She reports no difficulty breast-feeding for the first several days postpartum, but states that for the past week her daughter has had difficulty latching on. Three days ago her right nipple became dry and cracked, and since yesterday it has become increasingly swollen and painful. Her temperature is 38.3°C (101°F). Her right nipple and areola are warm, swollen, red, and tender. There is no fluctuance or induration, and no pus can be expressed from the nipple.
  1. continue breast feeding from both the breasts
  2. breastfeed from unaffected breast only
  3. immediately start antibiotics and breastfeed only when antibiotics are discontinued.
  4. pump and discard breastmilk till infection is over and then continue breatfedding
  5. stop breastfeeding immediately.
72. A 27-year-old woman presents to her obstetrician with the complaint of pain and swelling in her left breast. She reports a fever of around 38.3°C (101°F) for the past 2 days. She recently gave birth to a healthy baby girl and has been breastfeeding every 3-4 hours. Examination reveals focal tenderness just medial to the nipple with surrounding warmth and erythema. Her WBC count is 12,000/mm³
Which of the following is the best treatment:
  1. Amoxicillin
  2. Diclocloxacillin
  3. Penicillin v
  4. Erythromycin
  5. Levofloxacilin
3073. Sarita, a 30 year old woman develops a deep vein thrombosis in her left calf on fourth post operative day following cesarean section done for fetal distress. The patient is started on heparin and is scheduled to begin a 6 weeks course of warfarin therapy.
The patient is a devoted mother who wants to breast feed her baby.
Q. What is the advice which is given to the patient:
  1. Patient may continue breast feeding at her own risk.
  2. Patient should breast feed her baby only if her INR is at <2.5
  3. Patient can breast feed her baby after 6 weeks course of warfarin is over
  4. Warfarin is not a contraindication for lactation.
  5. Warfarin is absolutely contraindicated during lactation.
74. You are called to a maternity ward to see a 23 year old primi patient who had delivered a 2.7 kg baby boy 2 days back.She had a normal vaginal delivery and placenta delivered spontaneously.Now she complains of bloody vaginal discharge with no other signs.O/E you notice a sweetish odour bloody discharge on the vaginal walls and introitus.Sterile pelvic examination shoes a soft non tender uterus.Her P/R-78/min, B/P-110/76 mm of hg, temp-37*C,R/R-16/min. Her WBC count =10,000 with predominant granulocytes.What is the most appropriate step:
  1. Currettage
  2. oral antibiotics
  3. Reassurance
  4. Order urinalysis
  5. vaginal culture
75. A 30-year-old G3P2 woman with a history of hypertension presents to the birthing floor in labor. Following a prolonged labor and delivery with no fetal complications, she continues to bleed vaginally but remains afebrile. On bimanual examination, her uterus is soft, boggy, and enlarged. There are no visible lacerations. Uterine massage only slightly decreases the hemorrhage, and oxytocin is only mildly effective.
Q. Which of the following is the next best step in mgt:
  1. Dilatation and curretage
  2. PGF2A
  3. Methylergometrine
    31
  4. Misoprost
  5. Platelet transfusion
 
PHYSIOLOGICAL CHANGES IN PREGNANCY
76. The clotting factor which is not increased in pregnancy:
  1. Factor 2
  2. Factor 7
  3. Factor 10
  4. Factor 11
77. A prosthetic valve patient switches to heparin at what time during preg:
  1. 28 wks
  2. 32 wks
  3. 36 wks
  4. Post partum
78. Schwangershaft protein is the other name of APPG 2011:
  1. HCG
  2. Papp-1
  3. Pregnancy specific beta1 glycoprotein
  4. Activin
79. The following changes occur in urinary system in pregnancy except:
  1. Increased GFR (APPG 2011)
  2. Increased RBF
  3. Hypertrophy of bladder musculature
  4. Increased activity of ureters
80. Effect of PIH on GFR is:
  1. Incr GFR
  2. Decr GFR
  3. GFR remains the same
  4. GFR can incr or decr
81. Maximum teratogenecity occurs during:
  1. First two weeks after conception
  2. 3 – 8 weeks after conception
    32
  3. 8 – 12 weeks after conception
  4. 13 – 20 weeks after conception
82. Smallest diameter of fetal head:
  1. Bimastoid
  2. Bitemporal
  3. Occito frontal
  4. Submento vertical
83. During the delivery it is necessary to cut an episiotomy. The tear extends through the sphincter of the rectum, but rectal mucosa is intact. How would you classify this type of episiotomy?
  1. 1st degree
  2. 2nd degree
  3. 3rd degree
  4. 4th degree
 
DIAGNOSIS IN PREGNANCY
84. Ideal time to perform USG to measure nuchal translucency is…..wks of gestation
  1. 8-10 weeks
  2. 11-13 weeks
  3. 14-16 weeks
  4. 18-20 weeks
85. A 32 year old woman is 9 weeks pregnant and has a 10 yr old Downs syndrome child. W hat test would you recommend for the mother so that she can know abt her chances of getting Downs syndr baby in this present pregnancy.How will you assure her abt chances of Downs syndrome in the present pregnancy.
  1. Blood test
  2. USG
  3. Chorionic villi sampling
  4. Assure her that there is no chance since she is less than 35 years.
86. All of the following can be diagnosed by chorionic week sampling except:
  1. Trisomy 21
    33
  2. Cleft palate
  3. Gauchers disease
  4. Phenylketonuria
87. Most common complication of chorionic villi sampling done at 4 weeks is:
  1. Fetal loss
  2. Infections
  3. Limb reduction defects
  4. Bleeding
88. Minimum hcg level at which gestational sac can be detected by TVS is …..milli IU/ml
  1. 500
  2. 1000
  3. 2000
  4. 4000
89. A 17 year old comes to an adolescent clinic with complain of nausea and vomiting.She did a home urine pregnancy test which was positive. She does not remember her date of last menstrual period.USG shows a viable pregnancy of 8 weeks gestation.
Q. Which of the following statements regarding first trimester ultrasound is correct:
  1. A gestational sac can be first seen 2 weeks after LMP.
  2. The accuracy of determining gestational age using ultrasound begins to decrease after first trimester.
  3. Yolk sac is the first sign of pregnancy on USG
  4. USG can be used to determine the sex of the baby
90. A patient present for her first initial OB visit after performing a home pregnancy test and gives a last menstrual period of about 8 weeks ago. She says she is not entirely sure of her dates, however because she has a long history of irregular menses. Which of the following is the most accurate of way of dating the pregnancy
  1. Determination of uterine size on pelvic examination
  2. Quantitave serum HCG levels
  3. Crown rump length on abdominal or vaginal examination
  4. Determination of progesterone level along with serum HCG level
3491. Increased AFP is seen in:
  1. Downs syndrome
  2. Molar pregnancy
  3. Overestimated gestational age
  4. Congenital nephrotic syndrome
92. The finding of a single umblical artery on examination of the cord after delivery is:
  1. insignificant
  2. Occurs in 10% of newborn
  3. An indicator of considerably increased incidence of major malformations of the fetus.
  4. E qually common in newborn of dibetic and non diabetic mothers
93. The use of the following drug during pregnancy can lead to mobius syndrome:
  1. Warfarin
  2. Phenytoin (AIPG2012)
  3. Mifepristone
  4. Misoprostol
94. A syndrome of multiple congenital anomalies including microcephaly, cardiac anomalies and growth retardation has been described in children of women who are heavy users of:
  1. Amphetamines
  2. Barbiturates
  3. Heroin
  4. Methadone
  5. Ethyl alcohol
95. A 28 yr female G2P1 presents to antenatal clinic at 24 weeks for routine check up. USG shows a normal for gestational age fetus at 24 weeks of gestation in frank breech position, with no other abnormalities. What is the most appropriate next step in mgt:
  1. Glucose challenge test with 50 gms of glucose
  2. culture for Neisseria gonorhhea and Chlamydia trachomatis (normally done at initial visit and in certain high risk grps at 32–36 weeks along wid grp B streptococcal screening)
    35
  3. ECV
  4. immediate LSCS
  5. immediate induction and vaginal delivery
96. A 27-year-old G1P0 woman at 37 weeks’ gestation comes to the clinic because she is worried that she has not felt her baby moving for 2 days. She denies any recent trauma or abdominal pain. There has been no leaking of fluid, vaginal bleeding, or contractions. The woman underwent a non-stress test (NST) to test for fetal activity and was found to have a baseline rate of 115 bpm.
Q. Which of the following results would make one concerned about hypoxemia?
  1. decelerations at the beginning of contractions
  2. FHR incr to 140 bom several times during 5 mins
  3. FHR incr to 170 bpm on 2 occasions and returning to baseline in a pd of 20 mins
  4. FHR ranging from 110-120 bpm over a period of 40 mins
97. Fetal anemia leads to:
  1. early decelerations
  2. variable decelerations
  3. sinusoidal heart rate pattern
  4. accelerations.
98. A 27 yr old G3P2002 who is 34 weeks gest. age feeling the decreased fetal movements (once/hr) for past several hrs. She is healthy, has had regular prenatal care, and denies any complications till date.
1. How will you counsel the patient.
  1. Instruct the patient to go to labor and delivery for a contraction stress test
  2. Reassure the patient that one fetal movement per hr is within normal limits and she does not need to worry
  3. Counsel the patient that the baby is probably sleeping and that she should continue to monitor fetal kicks. If she continues to experience <5 kicks/hr by morning, she should call you back.
  4. Instruct the patient to go to labor and delivery for a nonstress test.
3699. The patient recieves a score of 8 on her biophysical profile. How should an obstetrician now counsel her
  1. The results are equivocal, and should have a repeat BPP within 24 hr
  2. The results are abnormal and she should be induced
  3. The results are normal and she can go home
  4. The results are abnormal, and she should undergo emergent CS
  5. The results are abnormal, she should undergo umbilical artery doppler velocimetry
100. A 26 yaer old primi patient presents to the prenatal clinic for the first time at 26 weeks of gestation.She reveals she has been taking cigratte daily and has habit of taking marijuana almost once in 2–3 days. Her past medical history is significant of chlamydia infection-USG (shown below Fig. 1) herniation of cerebellum thru foramen magnum and frontal bossing and lumbosacral kyphosis-Which of the following steps cud have prevented this condition
  1. Cessation of marijuana
  2. Cessation of smoking
  3. Taking vitamins before conceiving and continuing them after conceiving
  4. Early t/t of chlamydia
    zoom view
    Fig. 1:
37
 
TRUE /FALSE TYPE OF QUESTION
101. SLE can be managed during pregnancy using prednisolone, sulphsalazine and methotrexate.
102. I nternal podalic version is done under GA
103. A 31 year old pregnant female with twin gestation has more chances of Downs syndrome than general population
104. Hayman and Cho square sutures are used for managing PPH.
105. Placenta previa mostly needs cesarean section if placental edge lies within 2cms of the os…
106. PPH has a tendency to recur in subsequent pregnancy…
107. Direct coombs test detects maternal Ig M on fetal red cells…
108. Listeria infection may be suspected if meconium is present in amniotic fluid in < 34 weeks gestation…
109. DOC for pneumocystis carinii in pregnancy is pentamidine…
110. Peak systolic velocity is increases in middle cerebral artery with fetal anemia..
111. A 30 year old female with BMI 28kg/m2 should gain 14-16 kg weight during pregnancy….
112. Montevideo unit is contraction in mm of H2O per 10 mins…
113. M/c cause for second trimester recurrent abortions is cervical incompetence …
114. Trial of labour is C/I in previous LSCS pts….
115. Pelvis with AP diam more than transverse diam is Android pelvis…
116. Loss of knee jerks occurs when Mg SO4 conc->8 meq…
117. First episode of asymptomatic bacteriuria.The risk of having pyelonephritis is 15%….
38Pre NEET Obstetrics and Gynaecology
Answers
1. The answer is (b) i.e Middiastolic murmur
2. The answer is (c) i.e Aorti c aneurysm
3. The answer is (c) i.e 14 wks
4. The answer is (b) i.e Mitral stenosis
5. The answer is (a) i.e NYHA class > 3
 
 
Heart Disease in Pregnancy
Normal findings in CVS during pregnancy:
  • Pulse Rate increases
  • Diastolic B.P decreases
  • First heart sound is prominent and split
  • Second heart sound–normal
  • Third heart sound–normally not heard but in pregnancy it is prominent
  • Murmurs-ejection systolic murmur heard normally in aortic or pulmonary area at 10–12 weeks due to expanded intravenous volume.
  • Continous murmur heard normally over the tricuspid area in left 2–3rd intercostal space.
  • Apex beat is heard in the fourth ICS 2.5 cms left to midclavicular line
  • Slight cardiomegaly
  • Ecg- left axis deviation
39Indicators of Heart Disease during Pregnancy:
  • Systolic murmur greater than grade 3
  • Diastolic murmur
  • Marked Cardiomegaly
  • Sustained arrhythmia
  • Persistent split second heart sound
 
Most Common Heart Disease
zoom view
 
Clarkes Classification of Heart Disease in Pregnancy
Group I-Minimal risk (Mortality 0-1%)
Group III (Mortality 25-50%)
• ASD, VSD, PDA (congenital heart diseases)
• Fallot tetralogy (corrected)
• Any disease involving Pulmonary and tricuspid valve
• Bioprosthetic valve replacement
• Mital Stenosis belonging to class I, II according to NYHA
• Pulmonary hypertension -primary or secondary, an example of secondary being- Eisenmenger syndrome
• Marfan syndrome with aorta involvement (> 40 mm)
• Coarctation of aorta
NOTE: There is no need to remember Class 2 of the Clarkes classification.
40
 
Predictors of Cardiac Event during Pregnancy
Potential for an adverse cardiac event in a pregnant female as pulmonary edema, sustained arrhythmia, stroke, cardiac arrest or cardiac death can be estimated by following parameters.
N
New York Heart Association (NYHA) class >2
O
Obstructive lesions of the left heart (Mitral valve or aortic valve area <1 cm2).
P
Prior cardiac event before pregnancy—Heart failure, arrhythmia, transient ischemic attack, stroke
E
Ejection fraction <40%
The risk of cardiac complications is 3%, 30% and 60% when none, one or more than one of these complications are present.
NYHA classification (revised 1979)
  • Class I: No limitation of physical activity
  • Class II: Slight limitation of physical activity
  • Class III: Marked limitation of physical activity
 
Important Points to Remember in Heart Disease in Pregnancy
  • Time of hospitalisation in:
    • – Class I of NYHA – 36 weeks
    • – Class II 28 weeks.
    • – Class III and IV – If seen in the first trimester.
MTP should be advised ideally but if patient wants to continue pregnancy, then the women are hospitalized for the remainder of the pregnancy.
Remember: In India MTP is normally legal uptil 20 weeks but in heart disease patients MTP should not be done beyond 12 weeks as after 12 weeks the risk involved with delivery and abortion are the same.
  • Intrapartum Management:
    • – Patients should be allowed to go into spontaneous labour, if required induction with vaginal PGE2 may be done (Induction is safe in case of heart disease).
      Ref. Williams Obs. 22/e, p 1021-1022, 23/e, p 962
    • – Trial of labour is contrain dicated in patients of heart disease.
      41
    • – Vaginal delivery is preferred with the use of outlet forceps.
      Note in heart disease patients-even if there is no fetal distress, maternal distress or prolonge second stage of labour still we use forceps, this is called as prophylactic use of forceps.
    • Heart disease where vaginal delivery is contraindicated /Ceasrean section should be done are all those diseases where Aorta is involved (because in a patient with involved aorta, bearing down can lead to aortic rupture).
    • – Coarctation of aorta
    • – Aortic Aneurysm
    • – Marfans syndrome with aortic involvement.
    • – A patient who is fully anticoagulated with warfarin at the time of labor needs to be counseled for cesarean section because the baby is also anticoagulated and vaginal delivery carries increase risk to the fetus of intracranial hemorrhage.
      Anaesthesia given for LSCS due to heart disese: Epidural anaesthesia.
    • – If cesarean is being done for intracardiac shunts or aortic stenosis, GA is given to prevent hypotension.
      Antibiotic prophylaxis is given for prevention against infective endocarditis:
Recommendation of American College of Cardiology/American Heart Association for endocarditis prophylaxis regimens. (AHA 2007)
  • The American Heart Association recently updated its guidelines regarding which patients should take a precautionary antibiotic to prevent infective endocarditis (IE).
  • Prophylactic antibiotics are no longer recommended for gastrointestinal or genitourinary tract procedures. This recommendation follows from the observation that most cases of IE result from bacteremia caused by routine activities such as chewing food, brushing teeth, and flossing.
It is recommended that IE prophylaxis may be given during labor in the following subgroups of patients:
  • Prosthetic cardiac valve
  • Previous IE
  • Unrepaired congenital heart disease (including palliative shunts and 42conduits)
  • Completely repaired congenital heart defect with prosthetic material or device, during th first 6 months after the procedure.
  • Repaired congenital heart disease with residual defects at the site or adjacent to the site of aprosthetic patch or device.
  • Cardiac transplantation recipients who develop cardiac valvulopathy.
Antibiotic Regimen for IE Prophylaxis:
  • Only a few regimens are recommended by the American College of Obstetricians and Gynecologists (2008) 10 for prophylaxis which is given preferably 30-60 minutes before the procedure. Either I.ampicillin, 2 gm, or cefazolin or cef­tria­xone, 1 gm, is given intravenously.
  • For penicillin sensitive patients, one of the later regimen is given, or if there is history of anaphylaxis, then clindamycin, 600 mg is given intravenously. The recommended oral regimen is 2 gm of amoxicillin.If Enterococcus infection is of concern, vancomycin is also given.
Adequate pain relief (best by epidural anaesthesia). The Immediately After the Baby is Born
zoom view
43Contraception in Heart Disease:
  • Contraception of choice: Temporary–Barrier contraceptives (condoms)
  • Contraception to be avoided:
    1. OCPs(can precipiatate thromboembolic event)
    2. Intrauterine devices (can lead to infection)
Contraception of Choice: Permanent:
  • If the heart is not well compensated, the patient's husband is advised for vasectomy.Q
  • If heart is well compensated–tubal sterilisdation can be carried out:
    Sterilisation should be considered with the completion of the family at the end of first week in the puerperium under local anaesthesia through abdominal route by minilap technique.
Prognosis of Heart Disease in Pregnancy
General fundae
zoom view
Important Points:
  • Best time for cardiac surgery in Mitral Stenosis 14–18 wks
  • Surgery of choice Balloon valvuloplasty
  • Septic abortion m/c valve affected is tricuspid valve
  • M/C fetal complication in heart dis IUGR
  • M/C time of heart failure
    Immediate post partum>at the time of delivery>at 30–32 weeks of gestation.
44
 
Case Study
The diagnosis of peripartum cardiomyopathy should be kept in mind in all such cases.
The criteria for diagnosis are:
  1. Cardiac failure within last month of pregnancy or within 5 month postpartum.
  2. No determinable cause for failure (may be immunological or nutritional).
  3. No previous heart disease.
  4. Left ventricular dysfunction (Echocardiography) as evidenced by ejection fraction < 45%
  5. Left ventricular end-diastolic dimension > 2.7 cm/m2.
Predisposing Factors:
  • Multiparous
  • Young 20-35 years
  • Twins pregnancy
  • Chronic hypertension pre-eclampsia
  • Prolonged tocolytic therapy.
Investigation:
  • Chest X-ray: Enlarged heart and pulmonary vascular redistribution.
  • Echo: Enlargement of all chambers of the heart (predominantly left heart) with decreased ejection fraction in left ventricle.
 
DIABETES IN PREGNANCY
6. The answer is (c) i.e congenital adrenal hyperplasia
7. The answer is (c) i.e Congenital malformations in fetus
8. The answer is (b) i.e start insulin.
9. The answer is (a) i.e 70-100 mg%
45
 
Diabetes in pregnancy can be
Gestational Diabetes
Over Diabetes
• Normoglycemic female develops diabetes in pregnancy due to insulin resistance (insulin resistance in pregnancy is maximum at 24-28 weeks and is mainly due to the effect of hormone Human placental lactogen)
• These females will thus have high sugar levels at or after approx. 24 weeks
• In diabetic patients high blood sugar levels lead to formation of free radicals which in turn lead to fetal malformations, now in gestational diabetic patients free radicals will be formed approx. after 24 weeks (i.e when blood sugar levels will rise)
• By 24 weeks almost the organogenesis is complete in the fetus so it does not lead to congenital malformation in fetus.
• Hyperglycemic female becomes pregnant
• Switch them from oral hypogly cemic to insulin as oral hypogly- cemic can cross the placenta
• These females have high sugar levels from Day 1 of pregnancy so free radicals are formed from Day 1 and thus it can lead to congenital malformations in fetus
Thus answer to Q7 is (c) i.e Congenital malformations in fetus as the question itself is saying patient is a confirmed as a case of gestational diabetes.
It is a product of nonenzymatic glycosylation of hemoglobin. It reflects average blood sugar in preceding 6 to 8 weeks. HbA1c should be <6 gm% during pregnancy for good glycemic control. High HbA1c during the first trimester is associated with increased risk of gross congenital malformations and during second trimester is associated with macrosomia (HbA1c <8.5 gm% risk of malformation is 3.4%, HbA1c > 9.5 gm% risk of malformation is 22%).
46
 
Diagnostic Criteria for Diabetes during Pregnancy
According to American Diabetes association the criteria for diagnosis of overt diabetes during pregnancy is:
  1. Random plasma glucose >200 mg/dl
  2. Fasting blood glucose >125 mg/dl
  3. Two or more abnormal values on 100 gm oral glucose tolerance test during pregnancy.
Tests done in Diabetes patients during pregnancy.
zoom view
 
Complications of Diabetes
Maternal
Fetal
Neonatal
• Infection
• PIH
• Polyhydramnios
• Preterm labor
• 35-50% chances of developing diabetes mellitus
• Hyperglycemia
• Macrosomia
• Shoulder dystocia
• Abortions/IUD/ stillbirth
Hypoglycemia
• Hypocalcemia
• Hypomagnesemia
• Hypokalemia
• Hyperviscosity syndrome (hyper- bilirubinemia and polycythemia)
• Prematurity/RDS
Thus Ans. to Q6 is Congenital adrenal hyperplasia as it is not a complication of diabetes, so the risk of developing it is same in diabetes and non-diabetes.
 
Screening for Diabetes During Pregnancy
Glucose Challenges Test:
  • Performed by orally administering 50g of glucose and measuring 47venous plasma glucose 1 hour later irrespective of previous meal.
  • Interpretation of result:
    • – Plasma glucose: Interpretation
    • – 140 mg/dl: Further testing by GTT required
    • – 140 mg/dl: Further testing not required.
    • – 200 mg/dl: No further testing required as values > 200 mg/dl confirm the diagnosis of gestational diabetes.
  • Time for screening: Between 24 and 30 weeks of gestation (patients at high risk should be screened between 18-22 weeks and if initial screenings is negative it can be repeated between 26 and 30 weeks).Ideally it should be performed in all pregnant females but all those who have average/high risk for diabetes should be screened.
    Average risk:
    • – Members of ethnic group with high prevalence of GDM,
    • – diabetes in first degree relatives
    • – age > 25 years
    • – overweight before pregnancy,
    • – weight high at birth
    High risk:
    • – Marked obesity
    • – strong family history of type 2 DM
    • – previous history of GDM
    • – History of stillbirth
    • – History of delivery of large baby(> 4 kg)
    • – glycosuria
    • – h/o unexplained neonatal death
    • – h/o congenital malformation
    • – polyhydramnios
    • – h/o traumatic delivey with associated neurological disorder in infant
    • – h/o > 3 spontaneous abortions.
    • – Recurrent monoliasis,
    • – age > 30 years
    • – impaired glucose metabolism
48Diagnostic Test-Glucose Tolerance Test:
  • Patients with abnormal screening test are followed by a 3 hour glucose tolerance test (GTT)
  • The test is performed with 100gm of glucose. Upper limit of normal for the 3 hour glucose tolerance test during pregnancy:
    • – Fasting: 95 mg/dl
    • – One hour: 180 mg/dl
    • – Two hour: 155 mg/dl
    • – Three hour: 140 mg/dl
  • If two or more of these values are abnormal: Gestational diabetes is confirmed.
  • If one value is abnormal: Increased risk of complications like macrosomia and preeclampsia - eclampsia. (Though gestational diabetes is not present)
  • WHO recommends use of glucose of 75 g GTT and only two samples to be withdrawn viz: The fasting and the 2 hour value.
Management of Diabetes in Pregnancy:
  • Diet: Medical nutrition therapy (MNT): It is the cornerstone of treatment of diabetes in pregnancy. Caloric requirement is 25-35 kcal/kg body weight/ day according to body mass index (Table). It is advisable to take 3 major and 3 minor meals so that there is no intermittent hypoglycemia and still ideal blood sugars are maintained.
    Body mass index(BMI) kg/m2
    Calories intake
    18.5-24.9 (Normal)
    30 kcal/kg/day
    16.5-18.4 (Underweight)
    35 kcal/kg/day
    25-30 (Overweight)
    25 kcal/kg/day
    40 (Morbid obesity)
    12 kcal/kg/day
    Diet composition should be 50-60% of carbohydrates, 20% proteins and 25-30% fats
  • Exercise: Planned physical activity for 30 minutes/day is recommended for all individuals capable of participating. Advising patients to walk briskly or do arm exercises while seated in chair for at least 10 minutes after each meal accomplishes the goal.
49This should be continued for atleast 2-3 weeks with the aim to achieve-the following metabolic goals:
Metabolic Goals during Pregnancy:
  • Premeal value- 70-95 mg/dl (Answer 9)
  • Post meal (2 hrs post parandial) 120 mg/dl
  • HbA1c-6%.
    • – If these goals are not achieved patient should be put on Insulin. (Answer 8).
    • – Oral hypoglycemics are not advised during pregnancy as they can cross the placenta and cause fetal hypo­glycaemia.The only oral hypoglycemic drug approved for use in pregnancy is Glyburide
 
Intrapartum Management
Time of delivery:
  • Low risk patients-wait for spontaneous labor till maximum 41 weeks
  • High risk patients-37 completed weeks-induce labor as IUD occurs mostly in last 2 weeks of pregnancy.
  • Mode of delivery- vaginal delivery
Indications of Elective LSCS in GDM Patients:
  • Macrosomia > 4.5 kg (for predicting macrosomia, shoulder width > 14 cm, EFW > 4.5 kg on ultrasound)
  • Demonstrable fetal compromise (Severe IUGR)
  • Bad obstetric history
  • Other obstetric indications
 
Monitoring GDM patients during spontaneous labor:
GDM patients controlled on diet:
  • Blood glucose is maintained between 80–110 mg/dl.Blood sugar monitoring 2 hourly, serum electrolytes 12 hourly, urine sugar, ketones 4 hourly
GDM patients controlled on insulin:
  • Nil per orally
  • Blood glucose is maintained between 80–110 mg/dl
  • Blood sugar monitoring 2 hourly
    50
  • Serum electrolytes 12 hourly
  • Urine sugar, ketones 4 hourly 5 unit insulin in 500 ml of 5% dextrose at rate of 100 ml/ hour
  • If blood sugar> 140 mg/dl-plain insulin to be given subcutaneously according to sliding scale
  • Sliding scale:
    • – 140–180 mg% – 4 units
    • – 181–250 mg% – 8 units
    • – 251–400 mg% – 12 units
    • – > 400 mg% – 16 units
    If blood sugar < 80 mg/dl, infuse 5% dextrose at rate of 100 ml/hour.
    Frequent fetal heart monitoring for high risk pregnancy.
10. The answer is (d) i.e 300 mg
11. The answer is (b) i.e Iron deficiency anemia
 
Anemia in pregnancy
Definition: World Health Organization (WHO) has defined anemia during pregnancy as hemoglobin concentration of less than 11 gm% and a hematocrit of less than 33%. CDC (Center for Drug Control) proposes a cut off point of 11 gm% in 1st and 3rd trimester and 10.5 gm% during 2nd trimester
Severity of Anemia:
According to ICMR, severity of anemia is graded as:
Mild degree
10-10.9 gm%
Moderate degree
7-10 gm%
Severe degree
Less than 7 gm%
Very severe degree
Less than 4 gm%
 
Iron Requirements During Pregnancy
Total amount of iron required during pregnancy is 1000 mg, i.e 4-6 mg/day which can be calculated as:
• Fetus and placenta require
300mg
• Growing RBC of the mother require
500mg
• Lost through sweat, urineand faeces
200mg
51
• Lost at the time of delivery
200 mg
• Amount of iron s aved d/t amenorrhea
300mg
So approximately (1200-300 =) 900-1000mg is required during pregnancy.
Question number 10 asks the amount of iron required by fetus during pregnancy- which is 300 mg as explained above.
 
Also Know
  • All parameters of iron metabolism decrease during pregnancy except for the two Ts viz total iron binding capacity and trans­ferrin levels, which increase.
  • No matter in what form iron is being taken–only 10 % of it is absorbed, which means in order to fulfill the requirement of 4-6 mg/day, approximately 40–60 mg of iron should be taken in diet daily during pregnancy. Which is not possible, this is the reason why Iron supplementation is absolutely necessary in pregnancy.
In National Anemia Control Program under Ministry of Health and Family Welfare, all pregnant women who are not anemic are given folifer tablet containing 100 mg elemental iron (salt-ferrous sulphate) along with 500 µg folic acid for at least 100 days.
  • Earliest /most sensitive indicator of iron deficiency: Decrease in the levels of serum ferritin.
  • M/c anemia in pregnancy: In developing countries it is Dimorphic anemia i.e both due to iron and folic acid deficiency.
  • Physiological anemia during pregnancy: The increase in plasma volume (30–40%) is much more than the increase in red cell mass (10–15%) during pregnancy, leading to apparent decrease in hemoglobin level called as physiological anemia of pregnancy.
  • Starts at 7th–8th weeks
  • Maximum by 32 weeks
  • Does not go below 11 gm% in 1st trimester, 10 gm% in 2nd and 3rd trimester. (The rise in RBC volume begins at 20 weeks continues till term Therefore, in 3rd trimester there is slight rise I in hemoglobin concentration).
  • This hemodilution during pregnancy serves to reduce maternal blood viscosity, thereby enhancing placental perfusion and facilitating nutrient and oxygen delivery to the fetus.
52
 
Diagnosis of Anemia
zoom view
  • In the question number 11 patient has Hb 7.4 gm%, Hct 22% and symptoms of early fa tigue, which indicate she is anemic. Her complete blood picture shows MCV and MCH low indicating microcytic anemia. Thus the D/D could be Iron deficiency anemia or Thallassemia.
  • But since in this patient Nestrofts test(screening test for thallasemia) is negative thallasemia is ruled out and hence diagnosis is confirmed as Iron deficiency anemia.
  • NESTROFT test is ‘naked eye single tube red cell osmotic fragility test’. In this test 2 ml of 0.36 buffered saline solution is taken in one tube and 2 ml of distilled water in another tube. A drop of blood is added to each test tube and both the tubes are left undisturbed for 20 minutes. Both the tubes are then shaken and 53held against a white paper on which a black line is drawn. Normally, the line is clearly visible through the contents of tube containing distilled water. If the line is clearly visible similarly through the contents of tube with buffered saline, the test is negative. If the line is not clearly visible the test is considered positive. The principle is that normocytic normochromic cells when put in hypotonic solution will undergo lysis wheras in thalassemia trait, the cells are microcytic and hypochromic which are resistant to hemolysis due to decreased fragility. It has 91% sensitivity and 95% specificity and the negative predictive value is 99%.7 NESTROFT test is only a screening test for thalassemia. The definite test is the estimation of HbA2 levels by high liquid performance chromatography. In thalassemia HbA2 levels are > 3.5%.
Management of Anemia in Pregnancy:
zoom view
 
Response to Oral Iron
Laboratory parameters:
5-7 days:
Increase in reticulocyte count to up to 5% first parameter to increase after oral iron
2-3 weeks:
Increase in hemoblobin level @ 0.8-1.0 gm/dL/week
Improvement in RBC indices – MCV, MCH, MCHC
6-8 weeks:
Hemoglobin level comes to normal level
Peripheral smear shows normocytic normochromic RBC's
Increase in serum ferritin level.
  • Parenteral iron- It can be given by either intramuscular or intravenous route.
Dose of parenteral iron is calculated as:
  • Body weight in kg × (Desired Hb – patient's Hb) × 2.21 + 1000 mg
  • 1000 mg is taken for complete restoration of the stores in patients with continuing blood loss otherwise 500 mg is adequate for patients whose blood loss has been arrested.
    OR
    54
  • Give 250 mg elemental iron for each gm of Hb deficit and add another 50% for replenishment of stores.
  • Intramuscular iron preparations available are iron dextran, iron sorbitol citrate complex.
  • Intravenous iron preparations available are iron dextran, iron sucrose, ferrous gluconate.
The rise in hemoglobin after parenteral therapy is 0.7- 1.0 gm% per week which is same as seen with oral iron therapy, thus the aim of giving parenteral iron is not rapid rise in Hb levels. The main advantage of parenteral therapy is the certainity of its administration.
The indications of parenteral iron are:
  1. Intolerance to oral iron
  2. Impaired iron absorption
  3. Chronic blood loss
  4. Gastrointestinal disorders which gets aggravated by oral iron-peptic ulcer disease, ulcerative colitis
  5. After 30 weeks period of gestation, parenteral iron is preferred as the compliance is 100%.
  • Blood transfusion: 1 unit of blood raises the Hb levels by 0.8-1 gm% within 24hrs.
Indications of Blood Transfusion:
  1. Severe anemia seen beyond 36 weeks of pregnancy
  2. Anemia due to active blood loss
  3. Refractory anemia
NOTE: The Hb levels at the time of delivery should be atleast 7 gm%.
12. The answer is (b) i.e Start category 1 ATT in 1stTrimester
 
Tuberculosis in pregnancy
Important Points:
  • Increased chances of relapse and flare of tuberculosis occurs in puerperium.
  • It is not an indication for termination of pregnancy
  • ATT can be given at anytime during pregnancy, including first trimester
    55
  • INH + Rifampicin are given orally for 9 months (INH resistance-Ethambutol).
  • Baby should be given prophylactic INH for 3months, if montoux is negative after 3 months, stop INH and give injection BCG
  • In active lesions breast feeding is C/I,ATT is not a C/I for breast feeding.
  • Pulmonary TB is not an indication for performing cesarean section
 
ATT C/I DURING PREGNANCY
zoom view
 
Mnemonic-K F C Always Surprising
13. The answer is (d) i.e rectovaginal group B streptococcal culture
 
Neonatal sepsis
  • Group B streptococci, Streptococcus agalactiae is a major cause of neonatal mortality and morbidity.
  • Neonates present with respiratory distress, apnea, hypotension.
  • ACOG recommends universal culture screening for rectovaginal GBS at 35–37 weeks
Prophylaxis Against GBS
Intrapartum prophylaxis indicated
Intrapartum prophylaxis not indicated
• Previous infant with invasive GBS disease
• GBS bacteriuria during present pregnancy
• Positive GBS screening culture during present
pregnancy unless LSCS is planned
• Unknown GBS status with any of the Following
  • - Delivery at <37 weeks
    - Amniotic member rupture >18hrs
    - Intrapartum temp >100.4F
• Previous pregnancy with positive GBS screening culture
• Planned cesarean del per formed in absence of labor or
memb rupture (regardless of maternal GBS culture status)
• Negative GBS vaginal and rec tal screening culture
56
 
Drugs Used in GBS Prophylaxis
zoom view
14. The answer is (a) i.e Current symptoms of genital pain and tingling
15. The answer is (d) i.e perform elective LSCS only if mother has active herpes at the time of delivery.
Herpes Simplex Virus infection in pregnancy:
  • Most common time of Mother To Child transmission is–at the time of delivery
  • ACOG doesnot recommend a routine screening for HSV
  • DOC during pregnancy-Acyclovir(safe in lactation also) × 7-10 days
  • ACOG recommends daily viral therapy at or beyond 36 weeks for women who have recurrences during pregnancy as it decreases the outbreaks at term and so decreased need for cesarean.
  • Cesarean section is indicated for women with active genital lesions or in patients having prodromal symptoms of herpes viz genital pain and tingling (ans 14). Cesarean is not indicated in women with a h/o HSV infection but no active genital tract lesion/prodromal symptoms at the time of delivery.
  • If no active breast lesions are present-patient can breast­feed.
  • Now with this background about HSV infection, let′s have a look at the options given in Q15.
  • Option a: Administer one dose of acyclovir if she has active genital herpes at the time of delivery-incorrect as acyclovir should be given for 7–10 days in case of active herpes infection.
    57
  • Option b: Administer prophylaxis with acyclovir from now and uptil delivery whether she has active herpes or not–again incorrect as we have to give acyclovir for 7–10 days, then stop and restart at 36 weeks of gestation.
  • Option c: perform elective LSCS even if mother is asymptomatic at the time of delivery. – again Incorrect
  • Option d: perform elective LSCS only if mother has active herpes at the time of delivery. – correct
16. The answer is (c) i.e Baby must be treated immediately after birth as chickenpox is serious in newborns.
17. The answer is (d) i.e administer varicella immunoglobulin to the patient.
 
Varicella infection in pregnancy
Important Points:
  • If varicella infection occurs during first half of pregnancy(m/c time of transmission13 to 20 weeks) – it results in congenital varicella syndrome.
  • Congenital varicella syndrome is characterized by chorioretinitis, microophthalmia, cerebral cortical atrophy, hydronephrosis and bone or skin defects.
  • Congenital varicella syndrome is an indication for doing MTP.
  • Congenital defects rarely occurs if varicella infection occurs after 20 weeks.
  • Neonatal varicella is characterized by pneumonitis, hepatitis and DIC.
  • Perinatal varicella exposure just before or during delivery poses a serious threat to newborns and so Varicella Immunoglobulin (VZIG) should be given to neonates of born to mothers who have clinical evidence of of varicella 5 days before and upto 2 days after delivery.
  • The use of VZIG decreases the chances of neonatal varicella and also modify the clinical course but it doesnot always prevent severe or fatal varicella.Expectant treatment with close observation, followed by prompt initiation of antiviral therapy on suspicion of neonatal varicella is recommended.
    58
  • DOC for treatment of infected pregnant mothers-i/v acyclovir
  • Let me explain Q 17 – here the question says a pregnant woman is exposed to chicken pox rash, she doesnot have chicken pox…so obviously we will not treat her or her baby with acyclovir.Now since the female herself doesnot have chicken pox so why to give VZIG to the infant, rather this female should be given prophylactic VZIG so that she doesnot acquire chickenpox.
Varicella Prophylaxis: Exposed pregnant women who are susceptible should be given Varicella IG within 96 Hrs of exposure to prevent or attenuate varicella infection.
18. The answer is (c) i.e The proper treatment of infants born to infected mothers includes the administration of hepatitis B Ig as well as vaccine.
19. The answer is (a) i.e within 12 hrs
 
Hepatitis in pregnancy
  • M/C after blood transfusion – Hepatitis C
  • M/C causing fulminant hepatitis – Hepatitis E
  • M/C causing maternal death – Hepatitis E
  • M/C leading to vertical transmission – Hepatitis B
  • M/C time of transmission – 3rd trimester
If mother is HBs Ag positive, fetus after delivery should be given-HBV immunoglobulin (0.5 ml i/m, within 12 hrs of birth – Ans. 18 with 19) followed by Hepatitis B vaccine within 12hrs and and then 1 and 6 months later. Efficacy of treatment 85–90%.
If mother is Hbe Ag positive chances of transmission of hepatitis to fetus are high whereas presence of Hbeab is protective (in 25% females even if Anti H Be is present, vertical transmission occurs).
All pregnant females should be screened for Hepatitis –Time for screening-1st prenatal visit and should be repeated in high risk patients in third trimester.
 
Screening Test is–HBs Ag antigen
Role of cesarean section in preventing hepatitis B infection is controversial.
59
 
Also Know
Infections in Pregnancy:
  • Most common-CMV
  • Most teratogenic-Rubella
  • M/C Time for rubella transmission to fetus-1st trimester (maxmimum-1to 4 weeks),absent transmission-beyond 20 weeks.
  • In Rubella –M/C single defect which occurs is –Sensorineural hearing loss
  • Heart defects seen in Rubella -PDA and Pulmonary artery stenosis
  • After rubella vaccine, pregnancy is contraindicated for 1 month.
  • CMV-Transmission can occur in any trimester
  • -Most severe infection occurs if transmission occurs in 2nd trimester.
  • CMV transmission can occur during vaginal delivery and breast feeding also
  • Primary infection – leads to 40% transmission
  • Recurrent infection – leads to 0.2%-2% transmission.
  • CMV Never Leads to Heart Defects in Fetus
  • M/C time for toxoplasma infection-3rd trimester
  • Maxmimum/most severely fetus is affected if fetal infection occurs in 1st trimester
  • Triad of toxoplasma-intracerebral calcification, chorioretinitis and hydrocephalus
  • Treatment-Spiramycin (prevents fetal transmission but it cannot treat fetal infection if it is present.)
  • Spiramycin + pyrimethamine and sufonamide combination is given to treat fetal infection and prevent further transmission.
20. The answer is (a) i.e Despite the potential for fetal infection, she may opt out from the test
 
Important points on HIV in pregnancy
Least Teratogenic Infection: HIV
  • M/C time of vertical transmission:
    1. Peripartum period
    2. During delivery
    60
  • Perinatal transmission 15–40%
  • Risk depends on following factors- maternal viral load, CD4 count (inversely related), vitamin A deficiency and chorio­amnionitis.
  • Screening adopted for HIV is universal.
  • Screening-Opt out screening (i.e patient can opt out from the test)
  • HIV testing is the first step towards PPTCT (Prevention of parent to child transmission) aimed at reducing the vertical transmission of HIV infection.
  • Screening test–ELISA a nd a positive or indeterminate test should be followed by a Western blot for confirmation.
  • Antiretroviral therapy is given to pregnant female for 2 reasons:
    1. for benefit of her own health
    2. for preventing Mother To Child Transmission (HAART throughout pregnancy reduces MTCT to <2%)
  • Relationship of CD4 count to development of opportunistic infections In a patient of HIV.
  • All pregnant women should be given ART for two reasons:
    1. For benefit of her own health– these females should continue with treatment during pregnancy and afterwards
    2. HIV positive pregnant women who do not have indication for antiretro-viral therapy should have ART prophylaxis to prevent mother to child transmission
      (HAART throughout pregnancy reduces MTCT to <2%)
  • Salient points of WHO's latest Nov 2009 guidelines which were revised from previous (2006) guidelines:
    1. Although antiretroviral therapy among asymptomatic HIV infected nonpregnant adults and adolesents is generally delayed until the CD4 drops below 350 cells/µL, all pregnant women should be offered treatment regardless of CD4 + Two unit or viral load to reduce MTCT
      For infants of mothers with HIV who are taking therapeutic ART for their own health, the duration of prophylactic ART has been increased irrespective of whether the infant is breastfeeding or not. For breastfeeding infants, it is recommended that daily NVP is instituted from birth until 6 weeks of age. For nonbreastfeeding infants: daily AZT or NVP from birth until 6 weeks of age is recommended.
      61
    2. It is recommended that antepartum ARV prophylaxis should be started in all women from as early as 14 weeks gestation (if she is not already on ARV) or as soon as possible when women present late in pregnancy, in labor or at delivary.
 
HAART Timings
(Ref. Williams 23/e, p 1251)
Condition
Begin HRT
If a female is already on HAART and becomes pregnant
Continue HAART-even in 1st tri-mester
If a female is not on HAART and doesnot require ARV for her own health benefit but for MTCT prevention
Begin HAART after 1st trimester-(at 14 weeks)
Patient comes in labor for first time and is not on HAART
Give Navirapine and Zidovudine drip
If pregnant female is HIV and
Treat for HIV but Zidovudine is avoided.
Hepatitis B positive
Give Interferon alphato female after delivery and to baby give HBIG and hepatitisB vaccine within 12 hrs of birth
  • If HAART is given throughout pregnancy it reduces MTCT (Mother to Child transmission) to <2% and in such cases vaginal delivery can be done as cesarean will not decrease further risk of transmission.
  • If mother is not properly covered with ARV: Elective cesarean section at 38 weeks should be done or if woman is on ARV and still viral load is > 1000 copies/ml then also elective cesarean section is done.
  • After rupture of membranes the advantage of reducing Mother to child transmission by LSCS is lost, hence emphasis is laid on elective cesarean at 38 weeks.
  • Drug supplied by NACO: Free of cost-NEVIRAPINE (both for mother and baby)
  • If vaginal delivery is being done: Artificial Rupture of Membranes/Forceps/Vaccum//Fetal Scalp Electrodes are contraindicated.
  • First born twin has more risk of infection than second of twin.
  • After delivery: Breast feeding is not C/I in developing countries. Decision regarding the type of feed, i.e. breastfeed or replacement 62feed, should be made during antenatal period itself depending upon whether replacement feed is Acceptable, Feasible, Affordable, Sustainable and Safe (AFASS): As mixed feed is associated with increased rate of HIV transmission as compared to breastfeed alone, it is suggested the weaning must be complete and abrupt after six months of breastfeed or earlier if replacement feed is AFASS.
  • Answer to case study: Women who have been receiving antiretroviral treatment for their HIV-1 infection should continue same treatment during pregnancy, intrapartum and postpartum period except for Efavirenz (EFV). NVP(Nevirapine) should be substituted for EFV, although exposure to EFV during pregnancy is not an indication for abortion for women who become pregnant while receiving an EFV-containing regimen and are in the first trimester of pregnancy.
NOTE:
  1. Women who are receiving EFV and are in the second or third trimester of pregnancy can continue the current regimen.
  2. NVP should be given in women with CD4 count of 200-350 cells/mm3: There are data to show that women with a CD4 count of > 250 cells/mm3 face a higher risk of severe hepatotoxicity with NVP.
21. The answer is (a) i.e VDRL in mother and baby
22. The answer is (c) i.e Penicillin
 
Syphilis in Pregnancy
  • Bullous leisons on the body of the infant and presence of periostitis suggests the diagnosis of congenital syphilis. The only option related to syphilis is VDRL.
    Congenital syphilis:
    • – Transmission of T pallidum across the placenta from a syphilitic woman to her fetus may occur at any gestational age but fetal damage occurs when transmission occurs after 16weeks.
    • – Thus adequate treatment before 16 weeks may prevent fetal damage
    • – Untreated infection leads to fetal loss in 40% cases, IUD, stillbirth and abortions.(stillbirths being more common than abortions)
63
Early Congenital syphilis
Late congenital syphilis
Residual stigmata
• Appears within first 2 years of life, M/C time is 2-10 weeks age
• Earliest manifes- tation-rhinitis/ snuffles
• M/C-bone changes- osteochondritis
• Periostitis
• Mucocutaneous leison
• Hepatospleeno- megaly
• lymphadenopathy
• Appears after 2 years of life.
• Subclinical in most of the cases.
• Features- inter- stitial keratitis
• Eighth nerve deafness
• Recurrent arthropathy
• B/L knee effusion k/a Cluttons joint
• Asymptomatic neurosyphilis
• Gummatous periostitis
• Hutchinsons teeth (Centrally notched widely spaced peg shaped upper cen tral incisor
• Mulberry molars
In asymptomatic infants:
  • If mother has been treated with penicillin in 1st/2nd trimester-
    No treatment for infant
  • If mother has not been treated/received treatment with penicillin in third trimester –
    Treat infant with penicillin
 
SYPHILIS TREATMENT during pregnancy
Ref. Williams 23/e, p 1238
  • Syphilis therapy during pregnancy is given to eradicate maternal infection and to prevent congenital syphilis.
  • Parenteral penicillin G remains the preferred treatment for all stages of syphilis during pregnancy.
  • There are no proven alternative therapies for syphilis during pregnancy Erythromycin may be curative for mother, but because of limited transplacental passage, it does not prevent all congenital disease.
Women with H/O penicillin allergy, first penicillin desen­si­ti­zation should be done and then followed by penicillin injection.
64
 
Drug of Choice
Infection
Doc in Pregnancy
Bacterial vaginosis
Metronidazole to patient only 1st trimester-clindamycin
Pneumocystis carinii
Sulphamethozole-trimethoprim
Typhoid
Third gen cephalosporins/azithromycin
Syphilis < 1 year >1year
Benzathine penicillin 2.4 million U.i.m sigle dose Benzathine penicillin 2.4 million U. i.m weekly x 3doses
Gon orrhea
Inj Ceftriaxone 125 mg i.m single dose or, Tab cefixime 400mg single dose or, Inj Spectinomycin 2g i.m single dose
Chlamydia
Azithromycin single dose or Amoxicillin 500 mg TDS x 7days 2nd choice-Erythromycin
Grp B streptococci
Penicillin 2nd best- Ampicillin Penicillin resistant-cefazolin
Malaria-prophylaxis
Treatment Resistant cases (mostly d/t P. falciparum)
Chloroquine Chloroquine Quinine + clindamycin (mefloquine is currently not recommended …Williams 23/e, p1228)
Appendicitis
Immediate appendicectomy
Red degeneration
Conservative mgt (no termination of pregnancy and no myomectomy)
Thyrotoxicosis
Propylthiouracil
23. The answer is (d) i.e rapid urine b hcg measurement
In the question patient is presenting with amenorrhea of 6 weeks and she has history of being sexually active…now all of you know the most common cause of secondary amenorrhea is pregnancy…so first rule it out by doing a rapid urine hcg test, i.e Urine pregnancy test and then do USG to see whether the pregnancy is intrauterine or extrauterine. (the question specifically asks which is the next step in manangement)
Hcg–Imp points and testing:
  • Beta unit is specific unit
  • Secreted by syntiotrophoblast
  • Functionally similar to LH and acts via plasma membr LH-Hcg receptors.
  • Earliest detected 6-8 days after conception
    65
  • Doubling time-48 hrs
  • Maxm levels seen at 70 days/10 weeks /1st trimester
  • Most sensitive test to detect hcg- RIA>IRMA
  • It disappears from circulation 2 weeks postpartum
Increased
Decreased
• Multiple pregnancy
• Ectopic pregnancy
• Erythroblastosis fetalis
• Missed abortion
• Molar pregnancy
• Impending abortion
• Fetus wid Down syndrome
• Fetus wid Edward syndr (tri-somy 18)
24. The answer is (b) i.e missed abortion
 
Types of Abortion
  • Threatened Abortion: It is a clinical entity where the process of abortion has started but has not progressed to a state from which recovery is impossible.
  • Inevitable Abortion: It is a clinical entity where process of abortion has progressed to a state from where continuation of pregnancy is impossible.
  • Complete Abortion: Here the products of conception are expelled en masse.
  • Incomplete Abortion Here the entire products of conception are not expelled but a part is left inside the uterine cavity.
  • Missed Abortion: When the fetus is dead and retained inside the uterus for a variable period, it is missed abortion
Abortion
Clinical picture
Size of uterus
Internal OS
Ultrasound
Threatened
Slight bleeding
Corresponds
Closed
Live fetus
Inevitable
Bleeding + pain +/- shock
Corresponds
Open with products of conception felt
Dead fetus
Incomplete
Bleeding + passage of clots
Smaller
Open with products of conception felt and coming out through os
Incomplete fetus
Complete
Bleeding stopped
Smaller
Closed
Empty
Missed
Brownish discharge/ Smaller slight bleeding
Closed
Dead fetus
6625. The answer is (a) i.e Lupus anticoagulant
 
Antiphospholipid Antibody Syndrome
There are 3 main antibodies formed in antiphospholipid syndrome.
  1. Lupus articoagulant
  2. Anti cardiolipin Antibody
  3. Biologically false positive syphilis test antibody
Lupus antico-agulant (M/C)
• It leads to throm-bosis in the blood vessels unlike its name which suggests it should bring about anticoagulation-
• Thrombosis involves whichever blood vessel, manifes-tations occur accordingly.
• When thrombosis occurs in Placental blood vessels-abortion/IUD/still birth/IUGR/PIH can occur
Diagnosis-increased
Aptt, increased
Russel viper venom
clotting time, increased kaolin clotting time
T/t-heparin and aspirin
26. The answewr is (c) i.e Consumptive coagulopathy with hypo­­fibrinogenemia
Dead fetus if retained for > 4–5 weeks, release thromboplastin which leads to DIC.(consumptive coagulopathy)
 
Obstetrical Conditions Leading to DIC
  1. Septic abortion
  2. IUD
  3. Abruptio placentae
  4. Amniotic fluid embolism
  5. Severe preeclampsia, eclampsia. HELLP syndrome
27. The answer is (a) i.e B hcg levels will be higher than normal.
In the given question patient is presenting late in her first trimester of pregnancy with complains of persistent vaginal bleeding, nausea, and pelvic pain. Physical examination is notable for a gravid uterus larger than expected for gestational age. Fetal heart tones are absent.
D/D of height of uterus larger than the period of gestation:
  • Wrong dates
    67
  • Twin pregnancy
  • Molar pregnancy
  • Concealed variety of Abruptio placenta
  • Polyhydramnios.
Twin pregnancy can be ruled out because it doenot explain persistent vaginal bleeding and moreover in twin/multiple pregnancy fetal heart tones are not absent…2 or more FHS are heard depending on the number of fetuses.
Concealed variety of APH doesnot occur in late first tri­mester. APH by definition means any bleeding which occurs after 28 weeks of pregnancy and uptil the birth of the child and hence it can be ruled out although absent fetal tones and Fundal height more than the gestational age are seen.
Polyhydramnios again can be ruled out since bleeding cannot be explained by it …so we are left with molar pregnancy which explainds all the findings.
28. The answer is (a) i.e Admit to antenatal unit for bed rest and betamethasone.
Before discussing the question lets first discuss a few general concepts about placenta previa and its management.
Placenta Previa Risk Factors:
  • Multiparity and increased maternal age
  • Previous H/O placenta previa 12 times more risk
  • H/O any previous uterine surgery 4 time more risk
  • Previous uterine currettage
  • Increased placental size
  • Succenturiate lobe
  • Smoking
Placenta Previa Management:
  • Never do P/V examination
  • Investigation of choice: TVS(transvaginal scan… surprised don't 68be- because in placenta previa P/V examination is contraindicated since our finger has to inserted inside the internal os inorder to know the exact location of the placenta, which in turn can lead to torrential haemorrhage but in Transvaginal ultrasound, the probe is never taken beyond the internal os, it is kept in the cervical canal and obviously there are no chance of disturbing the placenta).
  • Double set up examination (i.e Per vaginal examination in the operation theatre with all arrangements of cesarean section done) can be done in placenta previa.
Management options in a Case of Placenta Previa
Expectant management-(Called as Macaffee regime) Goal- is to carry pregnancy till term without putting mothers life at risk with an aim to achieve fetal lung maturity.
Active management-To termi nate pregnancy immediately irrespective of gestational age.
• No Active bleeding present
• Hemodynamically stable
• Gestation age <37 weeks
• CTG-should be reactive
• No fetal anomaly on USG
• If active bleeding is present
• Hemodynamically unstable/ shock
• Gestational age >37 weeks and patient inlabour
• Fetal distress present/ FHS ab sent
• USG shows fetal anomaly or dead fetus
NOTE: In expectant management-Patient is admitted for the rest of the pregnancy, inj betamethasone is given to hasten the lung maturity of the fetus and blood is crossmatched and kept ready just in case patient starts bleeding again.The expectant mana­gement called as Macaffee and Jahnson regime should be continued till 37 weeks, but if anytime during expectant management patient rebleeds, pregnancy should be terminated immediately.
  • Mode of delivery practically in all patients of placenta previa is–Cesarean section
  • Low transverse cesarean section should be done but if placenta is anterior give vertical incision…williams 23/ep773
  • Whereas according to Dutta 7/e, p249 cesarean section should be done in all cases where placental edge is within 2cms from internal os.
    69
  • Now with this background lets have a look at the question.
  • G1P0 woman at 29 weeks’ gestation presents to the emergency department complaining of 2 hours of vaginal bleeding, the bleeding recently stopped, her vitals are stable (temperature is 36.8°C (98.2°F), blood pressure is 118/72 mm Hg, pulse is 75/min, and respiratory rate is 13/min), FHS are present and reassuring i.e there is no fetal distress…
  • This means we will manage this patient expectantly and there is no need to immediately terminate her preg­nancy..ruling out options c and d
  • So now we have to choose between option:
    1. Admit to antenatal unit for bed rest and betamethasone. And option
    2. Admit to antenatal unit for bed rest and blood transfusion.
  • The patients Hb is 11.1, there is no need for immediate blood transfusion (ruling out option b), just crossmatch and arrange blood and give betamethasone for hastening lung maturity.
29. The answer is (a) i.e Emergent cesarean section
Now this question can be explained in 2 ways but answer still remains the same:
Expl 1: Patient is presenting at 32 weeks of gestation to the emergency department with a small amount of vaginal bleeding. She doesn't have any pain., this could be a case of placenta previa.. now since there is fetal distress, we will do active management and terminate the pregnancy immediately by doing a cesarean section.
Expl 2: In this question patient has experienced small amount of painless vaginal bleeding…but the fetal distress doesnot coincide with the amount of blood loss, so probably this small amount of blood loss is fetal in origin this is why it has led to fetal distress i.e it is a case of vasa previa.
Management of vasa previa-Emergency cesarean section.
Vasa Previa:
  • It occurs due to velamentous insertion of the cord
  • Blood loss which occurs is fetal in origin and so there is increased fetal mortality – 75 to 90%, maternal mortality is not increased.
    70
  • Can be diagnosed antenatally by Doppler study
  • When bleeding occurs: Sinusoidal fetal heart rate pattern seen
  • Diagnosis made at the time of bleeding by-Singers alkali denaturation test/apt test
  • Management- Emergency Cesarean section.
30. The answer is (a) i.e DIC
31. The answer is (d) i.e release of thromboplastin by damaged placenta
32. The answer is (c) i.e administer fresh frozen plasma.
Abruptio Placenta-Important points:
  • Classification of Abruptio placenta is called as sher/page classification
    Management: Once Abruptio is diagnosed you have to manage it actively irrespective of the gestational age.
  • In case of abruption: The abruptio delivery interval is important.
  • Do not prolong this interval as complications like DIC/Renal failure (acute cortical necrosis) can occur.
  • Never give tocolytics in patients of abruptio (no matter how tempted you may feel)
  • Pritchard rule for management of abruption is Keep hematocrit atleast 30% and maintain urine output-30ml/hr
 
Mode of Delivery
zoom view
 
DIC
  • Release of thromboplastin in placental abruption leads to DIC. (ans 30 and 31… patient is 34 weeks preganat and involved in a car accident following which she starts bleeding per vaginally and 71has pain in abdomen also… i.e patient has abruption placenta and DIC is its complication which occurs due to release of thromboplastin)
  • In managing DIC – use fresh frozen plasma-1unitof FFP raises – 5–10 mg/dl of fibrinogen.
  • Cryoprecipitate – also increase fibrinogen but volume is not replenished
  • Platelet should be given if count < 50,000/ml. Single unit transfusion raises platelet by 5000-10,000/ml. If Female is Rh negative give 300 mcg of anti d after platelet transfusion
  • Side by side in abruptio delivery should be done
  • Uncorrected DIC is a contraindication for vaginal delivery/LSCS. (ans 32…in the question patient is having abruption with elevated fibrin degradation products which means she has DIC, which should be corrected prior to delivery by giving fresh frozen plasma.)
 
Normal Values of DIC Profile
  • Fibrinogen 150 – 600mg/dl
  • PT-11-16 sec
  • PTT 22–37 sec
  • Platelet – 1.5 to 3.5 lac D dimer /mm3
  • D dimer – <0.5 mg/l
  • Fibrin degradation products <10 mcg /dl
  • In case of DIC: All clotting factors are consumed so levels of fibrinogen decrease, PT and PTT, FDP, d dimer all increase.
NOTE: A fibrinogen level less than 100mg/dl or sufficiently prolonged PT/PTT in a woman with surgical bleeding is an indication for FFP in doses of 10–15ml/kg
33. The answer is (e) i.e severe preeclampsia
Before answering this question lets first understand the various terminologies related to Pregnancy induced hypertension.
72
Pregnancy induced hypertension
Chronic hypertension Pregnancy
(Means-a normotensive patient has conceived and due to some placental pathology, her B/P increases)
Liter ally means a hyper tensive female has conceived
Preeclampsia
Gestational Hypertension
• Rise in B/P after
20 weeks of pregnancy.
  • +
• Proteinuria (>300 mg in 24 hr urine collection or >30 mg/dl in a random urine sample or > + 1 on dipstick.)
  • +
• B/P comes back to normal within 12 weeks of delivery
• Like Preeclampsia but no proteinuria is associated
• Rise in B/P before 20 weeks
  • +
• No proteinuria
  • +
• B/P doesnot come back to normal within 12 weeks of delivery.
 
Preeclampsia can Further be Divided Into
Mild preeclampsia
Severe Preeclampsia
B/P
≥ 140/90 mm of hg but less than 160/ 110 mm of hg
≥ 160/110 mm of hg
Proteinuria
≥ 300 mg in 24 hr urine collection or ≥+ 1 on dipstick but < + 3
≥ 5g in 24 hour urine collection or ≥ + 3 persis tently on dipstick.
Symptoms:
Visual symptoms
Absent
Present
Oliguria (< 500 ml of urine in 24 hours)
Absent
Present
Epigastric pain
Absent
Present
Headache
Absent
Present
Features like HELLP
Absent
Present
syndrome:
Hemolysis,
Absent
Present
Elevated liver enzymes
Absent
Present
Low platelet count
Absent
Present
Renal functiontest:
S uric acid
Normal
Elevated
S urea
Normal
Elevated
S creatinine
Normal
Elevated
IUGR
Not seen
seen
73
  • Eclampsia is seizure or unexplained coma in a patient with preeclampsia
  • HELLP syndrome is a variant of preeclampsia defined by following criteria:
    • Hemolysis identified by Burr cells and schistocytes on an abnormal peripheral smear, an elevated serum bilirubin (>1.2 mg/dl) or LDH level (>600IU/L), or a low serum haptoglobin.
    • Thrombocytopenia with platelets <100,000/microl is the most consistent finding in HELLP syndrome.
    • Elevated Liver function tests (i.e transaminases) greater than two times the upper limit of normal.
Now in Q 33 patient is presenting at 24 weeks of gestational age with BP 162/114 mm of hg and proteinuria +3, earlier her BP was normal as suggested by the lines that she has come for routine prenatal visit and her pregnancy has remained uneventful till now. This means it is a case of Pregnancy induced hypertension (either Preeclampsia or gestational hypertension), since she is having proteinuria also it rules out gestational hypertension and favours Preeclampsia.
The B/P of the patient even 6 hours after initial checking is 160/110 mm of hg and her proteinuria is +3 which shift the diagnosis to severe preeclampsia.
 
Coming on the Management of these Conditions
  • Always remember Pregnancy induced hypertension (Preeclampsia/Gestational hypertension) are raised BP conditions due to placental pathology(incomplete trophoblastic invasion) and always their definitive treatment would be to Terminate the preganacy and throw out the defective placenta.
  • Antihypertensive of choice for Chronic hypertension in pregnancy-Methyldopa
  • Antihypertensive of choice for Pregnancy induced hyper­tension(Preeclampsia/Gestational hypertension) Labetalol
  • Antihypertensive of choice for Hypertensive crisis is-Labetalol.
74
 
Antihypertensives in Pregnancy
Safe
C/I
• Labetalol
• ACE inhibitors
• Calcium Channel blockers
• Diuretics
• Hydralazine
• Reserpine
• Alpha methyldopa
• Loratidine
• Sodium nitroprusside+/-
 
Management of PIH/eclampsia
Mild PIH
Severe PIH
Eclampsia
• Antihyperten- sives-Role is +/-, no proven efficacy. Generally bed rest and some diet restrictions are done.
• Definitive management- (as discussed earlier will always be) termination of pregnancy Done at 37 completed weeks of pregnancy
• 1st step in management is seizure prophylaxis- MgSO4
• Antihypertensives should be given to decreas BP in a controlled manner without compro- mising the utero- placental perfusion. Aim-Systolic BP should be between-140 to 155 mmof Hg and Diastolic BP should be between 90 to 105 mm of Hg
• Definitive mgt-termination of pregnancy at 34 completed weeks
• 1st step in management Airway management
• Drug to control sei zures-MgSO4
• Anti hyppertensives to control BP
• Definitive manage ment-immediate termi nation of pregnancy.
  • Route of delivery: Patients in labor or with a favorable cervix can deliver vaginally, for rest of the cases LSCS is indicated.
7534. The answer is d initial inpatient evaluation followed by restricted activity and outpatient management.
  • In the question patient has past history of hypertension which was controlled on diuretics and ACE inhibitors prior to pregnancy.Till date her B/P was normal, she was not using any antihypertensive and now all of a sudden her BP is 142/84 mm of hg and proteinuria is 0.35g all this suggests a possibility of superimposed preeclamplsia on chronic hypertension.
  • In this situation since BP is not much raised falling in the category of mild preeclampsia and gestational age is 35 weeks, no need to induce labor (labor should be induced at 37 weeks in mild preeclampsia) i.e option b ruled out.
  • I/V Furosemide and hydralazine again are not justified in mild preeclampsia patients (Role of antihypertensives is controversial in the setting of mild preeclampsia) i.e options aand c ruled out.
  • Her pre pregnancy regime which consisted of a diuretic along with ACE inhibitor cannot be started as ACE inhibitors are contraindicated during pregnancy ruling out optione.
  • So we are left with option d-initial inpatient evaluation followed by restricted activity and outpatient manage­ment. Which is the most logical step.
  • Also know: Worsening chronic hypertension is difficult to distinguish from superimposed pre eclampsia. If seizures, thrombocytopenia, pulmonary edema, unexplained hemolysis or elevation in liver enzymes develop, superim-posed preeclampsia should be diagnosed. Monitoring trends in BP and urine protein may be helpful. A 24 hr urine calcium measurement may also be helpful in detecting preeclampsia, as levels of urine calcium are lower (< 195 mg total urine calcium in 24 hrs) in preeclampsia patients than in patients with hypertension alone.
35. The answer is (c) i.e. I/V MgSO4
Ref: John Hopkins manual of gynae and obs 4/e,p 186
  • In the question patient is presenting at 28 weeks with rise in BP and proteinuria which confirms her as a case of preeclampsia.
  • The only confusion is whether she is a having mild preeclampsia or severe preeclampsia because that has a bearing on the management also.
    76
  • Lets say this patient has severe preeclampsia:
    Points in favour of the diagnosis
    Points against the diagnosis
    • Severe headache
    • 24 hour urine collection – 5g protein
    • B/P-155/85 mm of hg
    • Proteinuria + 2
    Read for youself what John Hopkins has to say on this issue.
    “Severe preeclampsia is classified by the following criteria:
    1. BP during bedrest of >160 mm of hg systolic or >110 mm of hg diastolic or
    2. Proteinuria > 300 mg on a 24 hr urine collectioneven if BP is in the mild range”
      Ref. John Hopkins manual of obs and gynae 4/e, p186
  • So our confusion about BP is over…now the second confusion is – this patients dipstick result favours mild preeclampsia whereas 24 hr urine result favours severe preeclampsia.
    Again read for yourself what JH manual has to say on this issue:
    Preeclamptic patients often have a wide variation in urine protein values over time, possibly from renal vasos­pasm. Discrepancies between the random urine dipstick and 24 hr urine collection measurements have been well described. The 24 hr urine coolection, therefore remains the preferred measure for diagnosing preeclampsia ”.
    Ref. John Hopkins manual of obs and gynae 4/e, p186
  • Our patient is thus a confirmed case of severe preeclampsia, and should be first managed by giving Mg SO4 as a prophylaxis against seizures.
  • Since she is only 28 weeks pregnant we will not perform cesarean immediately and try to carry the pregnancy uptil 34 weeks.
36. The answer is (d) i.e USG of fetal kidneys (Ref. John Hopkins manual of obs and gynae-4/e, p189)
ACE inhibitors are not recommended in pregnancy due to severe fetal malformations and n eonatal renal failure, pulm onary hypolplasia and fetal death.So if a female has taken ACE inhibitors in pregnancy, all the above side effects should be ruled out.
37. The answer is (d) i.e Diabetes
Before discussing the question, lets quickly revise oligohydramnios and polyhydramn ios.
77Oligohydramnios:
  • Liquor: <200 ml, AFI<5 cms, single pocket<2 cms
  • Causes:
  • D: Drugs like Prostaglandin synthatase inhibitors, e.g. Indo­methacin
  • I: IUGR
  • L: Leaking following amniocentesis
  • Mein: Maternal conditions like high B.P (PIH)
  • P: Post dated pregnancy
  • P: PROM
  • A: Congenital Abnormalities-triploidy, Amnion nodosum
  • R: Renal conditions of fetus like renal agenesis
 
Polyhydramnios
Liquor > 2 litres, AFI>25, Single largest pocket >8 cms
 
Causes
No Need to MUG
MUG UP
Multifetal pregnancy, Twin pregnancy (due to increased urine output)
Chorangioma of placenta
Maternal Hyperglycemia fetal hyperglycemia polyuria polyhydramnios
Downs syndrome
Fetal swallowing defects like oesophageal atresia, fascial clefts, neck masses
Rh negative pregnancy
Anencephaly, open spina bifida as CSF leaks into the amniotic fluid and increasing its volume.
TORCH infections
Alpha Thallesemia
 
RH NEGATIVE PREGNANCY
38. The answer is (d) i.e. Give Anti D prior to her amniocentesis
39. The answer is (b) i.e. Repeat titre in 4 weeks
7840. The answer is (c) i.e. within 3 days of delivery of a Rh positive fetus
 
Rh Negative Pregnancy–An Overview
Rh isoimmunization occurs when a Rh negative woman's immune system is sensitized to Rh factor present on the surface of the fetal erythrocytes, stimulating the production of immunoglobulin G (IgG) antibodies. Ig G antibodies can cross the placenta during subsequent pregnancies in alloimmunized women and if the fetus is positive for the erythrocyte surface antigens, result in hemolysis of fetal erythrocytes and anemia.
The primary immune response in mother to the D antigen is weak and occurs over 6 weeks to 12 months. The initial antibodies produce in mother are of IgM type that does not cross the placenta, as a result first pregnancy is not typically at great risk. IgG become detectable within 6 month time. A second antigen challenge generates amnestic response that is both rapid and almost exclusive IgG which can cross the placenta, thus posing the greater risk of severe fetal disease.
The most common routes of maternal sensitization are via blood transfusion or fetomaternal hemorrhage associated with delivery, trauma, spontaneous or induced abortion, ectopic pregnancy or invasive obstetric procedureslrefore in all Rh negative mothers before performing invasive procedures like amniocentesis, Anti D should be given.(Ans 38).
 
Fetal Problems in Rh Negative Pregnancy
Ig G antibodies against Rh antigen can cross the placenta and destroy the fetal erythrocytes which carry Rh antigen.
zoom view
  1. Results in i.sinusoidal heart rate pattern.
  2. Increased Peak systolic velocity in middle cerebral artery on Doppler study)
This IgG antibody mediated hemolysis of fetal erythrocytes, known as hemolytic disease of the fetus and newborn (HDFN), can have a varied degree of manifestations which are the result of anemia 79and hyperbilirubinemia. In severe cases, hemolysis may lead to extramedullary hematopoiesis leading to hepatosplenomegaly, decreased liver function and ensuing hypoproteinemia, ascites and anasarca in the fetus. When associated with high output cardiac failure and pericardial effusion, this condition is known as hydrops fetalis.
So in Rh negative pregnancy antibodies formed in mother eventually harm the fetus.
In an Rh Negative female if we want to know whether antibodies have been formed or not:
Methods are:
  • Indirect Coombs test: It is done on maternal blood and if it is positive it indictes, Rh antibodies are formed in mother whereas a negative test indicates that Rh antibodies are not formed i.e isoimmunisation has not occurred.
Now here it is very important to understand one very important concept:
Anti D is given to all pregnant Rh negative mothers:
  • Principle behind giving Anti D is, that if anti D is given to a Rh negative mother and suppose due to anyreason fetal blood with Rh antigen enters mothers circulation, this Anti D will lead an antigen antibody reaction and fetal RBC will be destroyed before it could stimulate mothers immune system to produce Rh antibodies.
  • Thus the usefulness of administering Anti D is only before maternal Rh antibodies are formed, if maternal antibodies are already formed then there is no point in giving Anti D.
  • In other words Anti D should only be give if Indirect coombs test is negative.
  • If Indirect Coombs test is positive then donot give Anti D, now it is important to know hoe much these antibodies have affected the fetus.
  • Whenever in Rh negative mothers we want know, to how much degree fetus is affected:
    • – Do PUBS (perumbIlical blood sampling) and measure the hematocrit of fetal blood- but that is lil risky, so it should be the last resort.
    • – Since the degradation product RBC is bilirubin therefore when hemolysis occurs fetus has jaundice and this bilirubin is excreted via fetal urine into the amniotic fluid (this is the reason why 80amniotic fluid is golden yellow in colour in Rh negative isoimmunisation) so do amniocentesis and measure the amount- of bilirubin by doing spectrophotometric analysis and measuring the delta optical density 450 and plotting the results on Lileys chart…this is most common method.
    • – In case hemolysis has occured fetus becomes anemic and peak systolic velocity in middle cerebral artery increases on doppler. These days management is being based on it.
 
Now Coming to Management of Rh Negative Pregnancies
Rh negative women presenting for obstetrical care can be categorized in two different groups:
  1. Rh negative nonimmunized women.
  2. Rh negative immunized women.
 
Management of Rh Negative non Immunized Mother
zoom view
NOTE: Possibility of Rh sensitization during antenatal period is very small, thus -Indirect Coombs test is performed at 28 weeks before the administration of anti-D immunoglobulin.
After giving Anti D, antibody titre should be performed at regular intervals in the pregnant female.
After the antepartum administration of anti-D immunoglobulin, the antibody screening will detect anti-D antibodies in the patient's serum, but the titer should not be greater than 1:4 at term. An antibody titer greater than 1:4 at term most probably results from alloimmunization/81isoimmunisation rather than anti-D immunoglobulin administration and such females again should be dealt in the same way like other isoimmunised females i.e category ii.
The Rh negative female who remains unsensitized i.e her antibodiy titre is always lower than 1:4 during pregnancy and have received anti-D immunoglobulin antenatally should be administered anti-D immunoglobulin in the postpartum period only when the following conditions are fulfilled:
  1. Infant is Rh positive
  2. Direct Coombs test on umbilical cord blood is negative.
NOTE: Just like Indirect coombs test is done antenatally on maternal blood and if it is negative then only Anti Dis given similarly, Direct Coombs test is done on infants blood after birth and Anti D given to mother if DCT is negative
 
Management of Rh Negative Immunized Pregnant Women
Management in this category requires the determination of whether it is a first affected pregnancy or woman already had a previously affected pregnancy.
 
First Affected Pregnancy
If this is a females first affected preg nancy that means first time ever in her obstetric history she is getting ICT positive-in such cases the moment their ICT is positive, antibody titre should be done.
Antibody titre of 1:16 is called as critical titre which means fetus is definitely affected.Only in these patients i.e who have first immunized pregnancy, Rh antibody titers can be used to determine the risk of fetal anemia, not in females who in earlier pregnancies also had ICT positive or females with history of hydrops fetalis. The rationale being that the association between the antibody titers and fetal affection that exists in the first affected pregnancy is lost during subsequent gestations. Also, in majority of first immunized pregnancies the anti-Rh antibody concentration is low and rarely exceeds the critical level of most laboratories. The critical level is that level below which no death due to fetal hemolytic disease has occurred within 1 week of delivery:
82
 
Management of Rh negative immunised pregnancy
zoom view
83
  • Serum antibody titers are done in these women every 4 weeks until the titers are found to be at or above the critical level (1:16).If titres are above critical level, there is no further use of antibody titer and the pregnancy is further monitored by middle cerebral artery peak systolic velocity (MCA-PSV) or amniotic fluid bilirubin concentration.
  • If antibody titers remains below critical level up to 36 weeks, the patient should be delivered by elective induction between 38-40 weeks and the birth of unaffected or mildly affected fetus should be anticipated.
  • If there is sudden rise of antibody titers above the critical level after 34 weeks but before 37 weeks of gestation, amniocentesis is done to assess the fetal lung maturity. Pregnancy should be terminated if the lungs are mature, but if the lungs are immature and the bilirubin level is low (less than 0.5 mg/dl), the pregnancy should be allowed to continue as long as weekly amniocentesis shows fetal pulmonary immaturity and a low bilirubin concentration. Delivery is contemplated as soon as these fetuses achieve lung maturity.
 
Women with Previous Affected Pregnancy
After the first affected pregnancy, the ability to predict fetal anemia from the maternal anti-D antibody titers is lost and now these pregnancies should be monitored by MCA-PSV and amniotic fluid bilirubin concentration. In these patients pregnancy can be followed by
 
Middle Cerebral Artery Peak Systolic Velocity
It is a noninvasive tool for the diagnosis of fetal anemia. The principle behind this test is that there is increased velocity of blood flow in the anemic fetuses due to increased cardiac output in an attempt to enhance the oxygenation.
 
Amniotic Fluid Spectrophotometry (AFS)
It detects the presence and severity of fetal hemolysis and anemia. By the middle of the second trimester, amniotic fluid consists predominantly of fetal urine and tracheopulmonary effluent, thus the amniotic fluid bilirubin is also elevated if the fetal hemolysis is there. Amniotic fluid containing high levels of bilirubin, such as that found in fetuses with severe hemolytic disease, is yellowish-brown. This observation by Liley in 1961 led to the development of a method to predict the severity of fetal hemolysis. In this method amniocentesis is done and amniotic 84fluid is subjected to spectro photo-metric analysis and ΔOD 450 is measured and result plotted on lileys graph.
zoom view
Fig. 2: Lileys curve
Results fall into one of the 3 Zones of Liley curve.
Zone
Management
A. Zone 1 (i.e. fetus is mildly affected)
• Repeat amniocentesis at 4 weeks and delivery at term 38-40 weeks
B. Zone 2 (i.e. fetus is moderately affected) In the lower zone 2-anticip ated Hb level is 11-13.9 mg/dl, in the upper zone 2 it is 8-10.9g/dl
• Repeat amniocentesis at 1-2 weeks
C. Zone 3 (i.e. fetus is severely affected) anticipated Hb = < 8 g/dl
• Intrauterine transfusion if preterm or delivery at 34 weeks if fetus is and delivery at salvageable
Ref. Fernando Arias 3/e, p 366-367
85The main limitation of the Liley's curve is that it starts at 26 weeks of gestation and extrapolation of the lines to earlier gestational ages is inaccurate. Queenan have developed a curve for fetal assessment from 14 to 40 weeks, divided into 4 zones.
zoom view
Fig. 3: Queenan curve
Now with this detailed understanding of Rh negative pregnancy let us have a look at Q. 38:
Points worth noting are:
  • Primi patient with Rh negative blood group
  • She is 37 years old-elderly primi (>30 years) and has risk of Down syndrome (>35 years)
  • She is concerned about the risk of having a down syndrome baby at this age and so insists on having amniocentesis done.
    • Option a: Advise against amniocentesis as it will increase the risk of isoimmunisation –although the risk of isoimmunisation will definitely be increased but still I will not advise her against it seeing her age and her concern.
    • Option b: Follow Rh titres carefully and give Anti D if evidence of isoimmunisation is present. Come on just now I have explained in that Anti d should be given only if evidence of isoimmunisationt is absent.Thus this statement is absolutely wrong.
      86
    • Option c: Give Anti D at 28 weeks of pregnancy and after delivery if baby is Rh negative. If baby is Rh negative, no need to give Anti D.
    • Option d: Give Anti D prior to her amniocentesis: This is the most logical step which should be done in this case.
    • Option e: Give rubella vaccine as she is Rubella non immune: Now I don't need to explain that Rubella vaccine is contraindicated during pregnancy.
39. The answer is b i.e. Repeat titre in 4 weeks
In the same patient–blood grouping shows A negative, with an anti D antibody titer of 1:4- the most appropriate next step in the management –Since this patient is a primi patient and her antibody titre is 1:4 so we should follow it by doing a repeat titre after every 4 weeks.
If this pregnancy would not have been her first affected pregnancy, then amniocentesis i.e. option a would have been the correct response.
40. The answer is (c) i.e Within 3 days of delivering an Rh negative fetus
87A dose of 50 mcg is recommended for prophylaxis following sensitizing events up to 12 weeks of pregnancy. For all events after 12 weeks, 300 mcg of anti-D immunoglobulin should be given.
After delivery Anti D should be given within 72 hours of delivery of Rh positive fetus but can be given uptil 28 days of delivery.
The usual dosage of 300 mcg anti-D immunoglobulin is capable of neutralizing the antigenic potential of up to 30 ml of fetal blood (15 ml of fetal red cells) and prevents Rh allo­immunization in 90% of cases. In less than 1% of cases in which the volume of fetomaternal hemorrhage exceeds 30 ml, Kleihauer-Betke test is used to quantitate the volume of FMH and the appropriate amount of anti-D IgG (i.e. 10 mcg/ml fetal blood) is administered accordingly.
41. The answer is (c) Serum bilirubin > 5mg/dl
42. The answer is (b) 36 weeks
Cholestasis in Pregnancy:
  • It 2nd most common cause of jaundice in pregnancy(m/c being hepatitis in pregnancy)
  • It is characterized by accumulation of bile acids in liver.
Clinical Features:
  • Manifestation appears beyond 30 weeks (i:e in 3rd trimester) and occasionally in late 2nd trimester.
  • 1st symptom to appear is pruritis-which has a predilection for the palms and soles. (pruritis appears 3 weeks before lab findings)
  • Jaundice is sligh-t bilirubin levels rarely exceed 5mg.
  • Recurs in subsequent pregnancies and so OCP's are contraindicated in females with H/O cholestasis during pregnancy.
  • M/c in twin pregnancy.
 
Investigation
Rise in s. bile acids is the earliest and the most consistent change(best marker for chol;estasis is bile acids-All india 2011). There is a 10-100 fold increase in S. Cholic acid followed by S. Chenodeoxycholic Acid.
  • Serum bilirubin level increases and increase is always in direct bilirubin but bilirubin levels rarely exceed 5 mg.
  • Serum alkaline phosphtase is mildly raised (increase in alkaline phosphatase is seen in liver damage, so it is seen in cholestatsis, 88but because the enzyme is also released by placenta in normal pregnancy so measurements are less useful as a diagnostic tool)
  • SGOT/SGPT increased (in nearly 100% cases) rarely more than 250U/L
  • Liver biopsy (definitive diagnosis)–no necrosis, no inflammation
  • MGT-doc-ursodiol, corticosteroids,antihistaminics,vit k, choles­tyramine can also be used
  • Time of delivery 37–38 weeks but if jaundice is present (as in question 42) then patient should be delivered at 36weeks
    Now coming to the Question No. 41
  • A pregnant female with idiopathic cholestatic jaundice – is associated with
    Option a: Intense itching – True
    Option b: SGDT, SGPT < 60IU – True
    Option c: S. bilirubin > 5m g/dl – Incorrect
  • In Idiopathic cholestasis of pregnancy bilirubin levels are rarely more than 5 mg as supported by:
    “Bilirubin levels rarely exceed 5 mg%” Ref. Williams 22/e, p 1126
    “Hyperbilirubinemia, results from retention of conjugated pigments; but total plasma concentration rarely exceeds 4-5 mg/dl”.
    Ref. Williams 23/e, p1064
    “Hyperbilirubinemia occurs in 20% of women and is almost exclusively direct reacting Bilirubin levels are usually between 2-5 mg/dl.”
    Ref. Mgt of High Risk pregnancy S.S trivedi, Manju puri Jaypee publication – pg 356
    Option d: Markedly increased levels of alkaline phosphatase-Incorrect. In cholestasis of pregnancy alkaline phosphatase may be mildly elevated and is not markedly elevated
  • Thus both option c and d are incorrect but if I have to choose one option I would mark option ‘c’ as my answer:
    • – Based on the fact that alkaline phosphatase as such does not carry much significance in the diagnosis of cholestasis (whether it is mildly / Markedly elevated)
    “Alkaline phosphatase increases above the normal elevation but is not much helpful in diagnosis.”
Ref. Mgt of High Risk Pregnancy
Ref. S.S trivedi Manjupuri Jaypee pub – pg 356
89Extra edge: Pruritis of cholestasis during pregnancy and physiological pruritis gravidarum in pregnancy can be differentiated by –GST – Glutathione S.transferase – It rises in cholestasis, atleast 9 weeks before the increase in bile acid.
43. The answer is (a) i.e Immediate delivery
 
Remember
 
Acute Fatty Liver of Pregnancy /Acute Yellow Atrophy of the Liver
It is a rare condition occurring in third trimester (mean gestational age of 37.5 weeks) Q.
Aetiology:
  • It is associated with disorders of fatty acid transport and oxidation
  • Deficiency of LCHAD enzyme i.e. long chair hydroxyl aceyl coenz A dehydrogenase
  • Risk are increased in case of:
    • – First pregnancy, male fetuses, preeclampsia, maternal obesity and multiple pregnancy.
Histology: Pathology
  • Liver is yellow, soft and greasy
  • Swollen hepatocytes with central nuclei and cytoplasm filled with microvesicular fat
Collectively called as:
  • Periportal sparing.
  • Acute yellow atrophy.
  • Minimal hepatocellular
 
Clinical Features
  • Patients present in the third trimester (generally at 37 weeks) with nonspecific symptoms like nausea, vomiting, anorexia, vague abdominal discomfort and malaise.
    90
  • In many women, persistent vomiting is the main symptom.
  • This is followed by jaundice after about one week.
  • Acute fatty liver of pregnancy should also be suspected in any woman who presents with new onset nausea and malaise in third trimester.
  • Ascities is seen in all patients.
  • In 50% cases-features of preclampsia viz-hypertension proteinuria and oedema are present.
Investigations:
  • Liver function tests are abnormal:
  • Serum bilirubin is increased but less than 10 mg/dl
  • Increase in SGOT and SGPT
  • Alkaline phosphatase is increased (moderately)
  • Prothrombin time may be increased
  • Clotting time increased
  • Clotting factors viz serum fibrinogen levels are increased
  • In severe cases, there may be disseminated coagulation failure.
Renal Function Test:
  • increased S. creatinine (present in all patients)
  • increased S. uric acid.
  • increased S. Ammonia levels
Others:
  • Decreased level of glucose (hypoglycaemia)
  • Decreased platelet count
  • Decreased fibrinogen levels.
Management:
  • T/t of hepatic encephalopathy: Fresh frozen plasma, cryoprecipitate, platelets and blood.
  • Rapid delivery is essential (Elective termination at 37 to 38 weeks is preferred).
 
Complication
Maternal mortality 10-75% due to
  • Hepatic encephalopathy
    91
  • PIH
  • DIC
  • Renal failure
Transient diabetes insipidus occurs during the period of recovery (due to increased vasopressin concentration).
Fetal Prognosis:
  • Fetal prognosis is poor.
  • If the fetuses survive, they may later on develop a Reye like syndrome of hepatic encephalopathy and severe hypoglycemia due to the defect in beta fatty acid oxidation.
44. The answer is (c) i.e Both B2 agonist and inhaled corticosteroids are safe in pregnancy
 
Asthma in Pregnancy – Important Points
Asthma is the most common chronic condition in pregnancy and affects 3% to 12% of gestations.
  • It is more likely to deteriorate in women with severe asthma.
  • Exacerbations are most frequent between 24 to 36 weeks gestation and are most commonly precipitated by viral respiratory infections and non compliance with inhaled corticosteroid regimens.
  • Because asthma exacerbation can be severe, they should be treated aggressively in pregnancy.
  • Complications of asthma in pregnancy: Preterm labor, preeclampsia, LBW baby and slight increase in abruptio placenta
  • Severity of asthma correlates with FEV1 (the FEV1 ideally is > 80% of the predicted, FEV1 less than 1L or less than 20% of predicted value, correlates with severe disease and PEFR (normally it ranges between 380-550 L/min).
Management of Asthma in Pregnancy:
  • Mild asthma: inhaled beta agonist (albuterol preferred because of more human data on safety in pregnancy)
  • Mild persistent: Low dose inhaled corticosteroids (Budesonide preferred)
  • Moderate: Low dose inhaled corticosteroids and long acting b agonist (Salmetrol preferred)
    92
  • Severe: High dose inhaled corticosteroid and long acting beta agonist and oral steroids if needed.
  • PGF-2alpha is absolutely C/I in patients of Asthma
  • So if in an asthamatic patient PPH occurs drug of choice is PGE1.
  • In asthamatic patients DOC for cervical ripening-PGE2 (PGE2 is not contraindicated in asthamatics)
45. The answer is (e) i.e Thyroxine is safe in pregnancy and the dose of thyroxine would be increased during preg­nancy to avoid hypothyroidism, which may affect the baby adversely.
 
Hypothyroidism in Pregnancy
  • M/C cause-Autoimmune cause-Hashimoto thyroiditis
  • Hypothyroidism can lead to Mental retardation in baby, abortion, stillbirth, IUGR, prematurity
  • Since maternal Hypothyroidism in pregnancy (whether overt or subclinical) may impair featl neuropsychological development, hypothyroidism should be treated adequately in pregnancy.
  • Thyroxine requirement increase during pregnancy and this increased requirement begins as early as 5 weeks.
  • DOC for thyrotoxicosis in pregnancy–Propylthiouracil.
46. The answer is (b) i.e Division after formation of the em­­bryo­nic disk result in conjoined twins.
  • Twin preganancy can be monozygotic (uniovular) or Dizygotic (binovular).
  • Uniovular twinning occurs due to fertilisation of a single ovum by a single sperm followed by fission of single fertilized ovum. It is less common than binovular twins (i.e option a is incorrect).
  • Uniovular twins are always of the same sex and are identical in appearance, resembling each other both physically and mentally, and even at times showing the same pathological tendencies.
  • Uniovular or monozygotic twins can have a single placenta (monochorionic) OR can have separate placenta (dichorionic) depending on the time of division of the fertilized ovum.
93
 
Monozygotic Twins
Division of single fertilised ovum
< 3 days
4 – 8 days
8 days
14 days
dichorionic
monochorionic
monochorionic
conjoined twin
diamniotic
diamniotic
monoamniotic
  • Binovular is fertilization of two ova by two sperms. Such twins are more frequent than uniovular (3:1). They may be of the same sex or opposite sexes and show a degree of resemblance no more than that of brothers and sisters from different births, this is the reason they can be called as “fraternal twins”.
  • Binovuar or dizygotic twins always have 2 placenta, i.e they are dichorionic.
  • Thus dichorionicity can either be due to dizygotic or it could be due to monozygotic twins (i.e option d is incorrect)
  • The incidence of monozygotic twins is fixed throughout the world..1 in 250 deliveries, whereas incidence of Dizygotic twins varies from region to region, being maximum in nigeria. (i.e option c is incorrect)
  • Although multifetal births account for only 3% of all live births, they are responsible for a disproportionate share of perinatal morbidity and mortality. (i.e option e is incorrect)
  • There is a higher risk of low birth weight babies and preterm labor. Multiple pregnancies are commonly associated with moderate to severe anemia, gestational hypertension malpresentation, polyhydramnios, cord prolapse,abruption or placenta previa.
  • It's also associa1ted with abnormalities like discordance, twin reversed arterial perfusion (TRAP) or conjoint twins.
47. The answer is (d) i.e Dichorionic and diamniotic in monozygotic twins
  • As discussed dizygotic twins always have dichorionic and diamniotic placentas (i.e option a and c are incorrect)
  • Monozygotic twins can have monochorionic or monoamniotic placentas depending upon the time of division…but always remember- The amnion develops after the chorion, so dichorionicity implies diamnionicity … it can never be that a twin is dichorionic but monoamniotic (i.e option b is incorrect)
9448. The answer is (b) i.e Anemia
This scenario represents a typical case of twin to twin transfusion syndrome.
 
Twin to Twin Transfusion Syndrome
  • It is always seen in monochorionic placenta.
  • There is an AV malformation such that there exists a communication from the umbilical arterial system of the “donor” twin to the umbilical vein of the “recipient” twin.
  • The donor twin is growth restricted, hypovolemic, has oligohydramnios and is anemic becoz it gives blood to the recipient twin. The recipient is larger, hypervolemic, has polyhydramnios and is plethoric. It has also been termed as Twin Oligohydramnios/Polyhydramnios Sequence (TOPS).
  • The earlier the TOPS appears, worse will be the prognosis.
  • TTTS is more common in female fetuses.
  • Coming to the question- Twin A is the recipient twin and Twin B is the donor twin since it has oligohydramnios. Thus Twin A can have CHF- due to volume overload, will have hydramnios and can have thrombosis since it has polycythemia which can lead to thrombosis, but never will it have anemia.
 
Also Know
Diagnosis of TTTS: By injecting O negative leukocyte poor washed RBC into the smaller (donor) twin and immediately removing the blood from the cotwin and testing it with Kleihauer-Betke stain, twin to twin transfusion can be confirmed
  • Since Arterial supply from the donor twin drains into the venous system of recipient. Doppler can pick up the vessels from one twin's umbilical cord and then finding a continuing vessel with venous flow coursing towards the co-twin's umbilical cord.
 
Management
The TTTS can cause Preterm delivery due to Polyhydramnios, IUGR or fetal demise.
  • Amnioreduction has shown improvement in fetal survival in 65% of cases. When the amniotic fluid exceeds 40 cm, one liter of fluid is removed for every 10 cms rise above normal.95
  • This procedure takes care of polyhydramnios and fetuses are born later in gestation and weigh more than untreated cases with TOPS. (Twin oligohydramnios – Polyhydramnios sequence)
  • Amniotic septostomy can create artificial opening between the gestational sacs to produce in effect a monoamniotic pregnancy. TTTS is seldom seen in monoamniotic gestations. There is speculation that the donor twin has access to amniotic fluid via the opening so it can correct the oligohydramnios by oral rehydration.
  • The vessels on the placental surface can be ablated by neodymium:yttrium-aluminumgarnet laser -Severe twin–twin transfusion syndrome presenting before 26 weeks of gestation should be treated by laser ablation rather than by amnioreduction or septostomy.
  • Finally, selective fetoreduction (SFR) can treat TTTS but this method is reserved for refractory cases to other forms of therapy or when in utero death of one of the twins is imminent.
 
INDUCTION OF LABOR
49. The answer is (a) i.e indomethacin
50. The answer is (a) i.e atosiban
All the drugs given in Q. 49 can be used as tocolytic but indomethacin should not be used beyond 32 weeks as it can lead to premature closure of ductus ateriosus.
 
TOCOLYTICS
  • P–Prostaglandin synthatase inhibitor like indomethacin, sulindac
  • C–CA channel blocker-Nifedipine
  • O–Oxytocin antagonist-Atosiban
  • D–Diazoxide
  • Mein–Mg SO4
  • NO–Nitric oxide inhibitor-Glyceryl triniitrate
  • Periodic–progesterone
  • Bleeding–b mimetics-salbutamol, ritrodrine, isoxsuprine, terbutaline
96
 
Mnemonic
PCOD Mein No Periodic Bleeding
zoom view
 
Role of Tocolytics in Heart Disease
Most of them are contraindicated in pregnancy:
  • Safest tocolytic is atosiban (oxytocin antagonist)
  • Beta agonist is contraindicated in cardiac arrthymias, valvular disease and cardiac ischemia because of their sympathomimetic side effects such as tachycardia, palpitation and hypotension.
  • Nifedipine is contraindicated in conduction defect, left ventricular failure due to side effects as tachycardia, hypotension, etc.
51. The answer is (c) i.e complete placenta previa (Ref. John Hopkins manual of Obs and gynae-4/e, p 77)
  • Friends there are a number of conditions in which labor can be induced, so for all such Questions remember those conditions in which induction of labor is contraindicated.
  • Contraindications for induction of labor-All those condition in which cesarean section necessarily has to be done, are contraindications for induction of labor viz.
  • C–Contracted pelvis (Naegles pelvis/Roberts pelvis)/Advanced Cervical cancer
  • A–Active genital herpes infection/High viral load HIV
  • P–placenta previa/Vasa previa/Cord prolapse
    97
  • U–Uterine scar of classical cesarean section, myomectomy scar, hysterotomy scar
  • T–Transverse lie.
 
MNEMONIC-CAPUT
52. The answer is (c) i.e Early cord clamping
53. The answer is (b) i.e post dated pregnancy
 
Active Management of 3rd Stage of Labour Includes
  • Administering a uterotonic agent after the delivery of the shoulder of the baby.
    The uterotonic agent of choice is oxytocin followed by methy­lergometrine.
  • Delayed cord clamping, so that the blood in the cord (approx. 40–50 ml) goes to the baby and is not wasted.
  • Delivery of the placenta by controlled cord traction/Modified Brandt Andrews technique.
  • Massage of the uterus after the delivery of the placenta.
Active management of labor-reduces the duration of third stage of labor from 15 minutes to 5 minutes and thus reduces the amount of blood lost, so in turn reduces the chances of PPH and decrease maternal mortality.
The only disadvantage of Active management of labor is, since uterotonic agent is given before the delivery of placenta thus chances of Retained placenta are increased.
Normally in active management of third stage we do delayed cord clamping but in certain conditions early cord clamping is advocated.
Indications of Early Cord Clamping:
  • Preterm or growth restricted fetus due to risk of hypervolemia (even an extra 40-50 ml of blood can cause CHF in premature infants)
  • Birth asphyxia (first immediately resuscitate the baby and then think about anything else)
  • Rh isoimmunization
  • HIV positive female
  • Maternal diabetes.
9854. The answer is (c) i.e Flex mothers thigh against her abdomen.
This patient has shoulder dystocia as head initially progresses beyond the perineum and then retracts which is called as Turtle sign.
Turtle sign positive: Head delivers but retracts against symphisis pubis
Shoulder dystocia (difficulty in delivery of shoulders) Impaction of anterior shoulder of fetus against symphysis pubis after fetal head has been delivered. Occurs when breadth of shoulders is greater than BPD.
Risk factors are:
D – Maternal Diabetes
O – Obesity-maternal and fetal, i.e macrosomia
P – Post term baby
A – Anencephaly: In anencephaly pseudo shoulder dystocia occurs.
Complications: M/C complication is Erbs palsy, klumpkes palsy, fracture clavicle fracture humerus, fracture spine, PPH (due to Injury to perineum), Fetal hypoxia.
 
Shoulder Dystocia Drill
1st Line of management
2nd Line
3rd Line
• Stop giving fundal pressure
• Can give supra- pressure
• Best/most effective pubic pressure manouvre-Mc Roberts manouvre (flexion and abduction of thigh)
• Woods Corkscrew manouvre
• Cleidotomy (# clavicle of baby)
• Symphiosotomy (divide pubic symphysis of mother)
• Zavaneilli manouvre (push head back and do cesarean)
  • Going with the drill, we should first try 1st line steps and then others, this rules out option d i.e Push infants head back into the uterus and do cesarean section and e i.e do a symphiosotomy.
  • Fundal pressure should never be applied in case of shoulder dystocia ruling out option (b).
  • This automatically leaves us with 2 options (a) abduct mothers thigh and apply suprapubic pressure and (c) i.e Flex mothers thigh against her abdomen.
    99
  • Now this is common sense that while doing Mc Roberts manouvre we will first flex the patients legs, and then abduct them and not vice versa…so the immediate next step is flex her thighs, i.e option (c).
55. The answer is (d) i.e Speculum examination to rule out leaking and usually assess cervical dilatation and efface­­ment.
In the Q important points to note are:
  • Patient is coming in preterm labor-31 and has contractions every 8 mins lasting for 20–30 secs
  • She is not sure whether her water bag has broken or not(i.e unsure whether Preterm premature ruture has occurred or not)
  • Cephalic presentation and no fetal distress
Now they are asking–Which of the following is the next best step in management:
  • Cervical culture for group B streptococci – incorrect as the usual time for screening for GBS is at 36 weks, since this patient is in preterm labor and GBS status is unknown, we will not do culture and wait for the report…by that time she would have already delivered and infant already infected, so we will give her a shot of penicillin as a prophlaxis against GBS
  • Digital cervical examination and assessment of dilation and effacement- incorrect as patient isnot sure whether her bag has broken or not, just in case membranes have ruptured, digital examination is contraindicated.
In case of Preterm Premature rupture of membranes:
“Only sterile speculum examination should be performed. Avoid digital examination unless delivery is imminent. Digital examination decreases the latency period and increases the risk of neonatal sepsis.”
Ref. John Hopkins manual of obs and gynae 4/e, p128
  • Quantification of strength and timing of contraction with external tocometer- not necessary since we have already been given the frequency and duration of uterine contraction.
  • Speculum examination to rule out leaking and usually assess cervical dilatation and effacement-this is the logical next step in this patient. In all cases of preterm labor before performing digital examination sterile speculum examination should be done to rule out ruptured membranes and to assess cervical dilatation.
100Preterm labor:
  • “Sterile speculum examination- done to inspect visually for bleeding, amniotic fluid pooling,advanced dilatation, bulging membranesand purulent cervical discharge. After ruling out PPROM, perform digital examination for dilatation, effacement, and station.”
Ref. John Hopkins manual of obs and gynae 4/e, p124
  • USG examination of the fetus- Iknow most of you must have thought this option is the correct answer but always read the Q very carefully…they are asking the next best step in manangement—which will be Sterile speculum examination… Always remember not only for exams but also for your practice years, whenever a female come to you in labor, before knowing how much she is dilated donot send her for USG, because if she delivers on the way…next day you will be in news…!!!!!
56. The answer is (a) i.e Sedate the patient and wait
This patient is presenting at 37 weeks of gestation with loss of engagement, 1 cm effacement of cervix and 10 uterine contractions per hour.She is hemodynamically stable and not in distress.
So she could either be in false labor or it could be a case of prolonged latent phase of labor and the best way to distinguish both is to sedate the patient, if patient is in false labor when she will wake up her pains would have subsided and if she is having prolonged latent phase of labor, then when she wakes up she will automatically be in active phase.
 
Remember-1ST Stage of Labour
Latent phase
Active phase
• Cervical effacement
• Begins with true labor pains and ends when cervix is 3 cms dilated
• Normal duration-nulli-8hrs
• Multi-4-6 hrs (5.3hrs)
• Cervical dilatation
• Begins when cx is 3cms dilated and ends when cx is fully dilated
• Minm rate of dil-primi-1.2cms/hr
• Multi-1.5 cms/hr
• Descent begins when cx is 7-8 cms dilated
• Friedmann curve-phase of acceleration-phase of maxm slope-deceleration phase
10157. The answer is (c) i.e attempt forceps delivery
In the question points worth noting are:
  • Patient is in labor has been completely dilated and pushing for 3 hrs
  • She has taken epidural analgesia.
  • She is exhausted and her temp is 37.8^c (i.e maternal distress is present)
  • FHS is 170/min with decreased variability. (i.e fetal distress is present.
  • P/V shows cervix is fully dilated and fetal head is at +3 station.
  • This is a case of prolonged second stage of labor with fetal distress.
  • Second stage: Normal duration is 0-30 mins for multiparous and 2 hours for nulliparous.
  • Prolonged second stage is defined as:
    Nulliparous
    Multiparous
    >2hr
    >1hr
    Without epidural analgesia
    >3hr
    >2hr
    With epidural analgesia
  • In the question patient is fully dilated i.e she is in second stage of labor and is a primigravida who has taken epidural analgesia, has been pushing for 3 hrsand has still not delivered i.e prolonged second stage.
  • Management of prolonged second stage is either instrumental delivery-forceps/ventouse or Cesarean section.
Always Remember 2 Things:
  • Forceps are better than vaccum in fetal distress
  • In case of fetal distress, with full dilatation of the cervix and station of fetal head at or below +2 station- Forceps are much faster way of delivery than cesarean section.
Here since patient is fully dilated, station of fetal head is +3 and fetal distress is present, so we will opt for forceps.
58. The answer is (b) i.e monitor and follow labor on partogram
The points worth noting in the question are:
  • Patient is in labor for one hour
  • She belives her water bag has broken
  • She has H/O previous LSCS for fetal distress.
    102
  • All P/A examination findings are normal
  • Fetal heart rate tracing is normal. (baseline FHS 140/min, beat to beat variability is present, occasional heart rate acceleration of 160/min for 15–20 secs. There are also decelarations to 115–120/min with the onset of contractions- note decelerations at the beginning of contraction signify early decelerations are normal becoz when contraction begins, head of the baby is compressed producing decelaration but it recovers soon)
What is the most appropriate next step in management?
  • Augment contractions with oxytocin–personally I would not do it for two reasons
    • – She ia a previous LSCS patient
    • – Oxytocin should be given to augment labor when the patient is in active phase of labor. Avoid using oxytocin in latent phase as chances of fetal distress and cesarean are increased. (and I know she is still in latent phase probably because she is in labor for 1 hour only)
  • Monitor and follow labor on partogram- very logical step and should be done in all patients in labor irrespective of whether he is a previous cesarean section patient or not.
  • Obtain immediate consent for LSCS: Why will I do it??? There is no indication for doing cesarean in this patient as her previous LSCS was done for fetal distress not for any recurring cause like contracted pelvis.
  • Send the patient for BPS: Again no need as there is no fetal distress or any other indication for doing so.
  • Perform urgent aminoinfusion–no need, even if her membranes have ruptured, she is already in labour.
59. The answer is (a) i.e emergency cesarean section
Points worth noting here are:
  • Patient is presenting with C/O decreased fetal movements.
  • FHR = 180/min with absent variability (i.e fetal distress is present)
  • Uterine contractions are present-every 3 minutes
  • O/E-Cervix is long /closed/-2 station.
  • Friends always remember–The earliest indication of fetal distress is 103known by the mother–whenever a pregnant female complains of decreased fetal movements, do not ignore it- rule out fetal distress and then send the patient back home.
  • In this case patient is complaining of decreased fetal movements and fetal heart rate is 180 i.e fetal distress is present so. Option a–i.e emergency cesarean section should be done. Since cervix is not favourable and fetal distress present we cannot wait for vaginal delivery.
 
Also know
Fetal kick count- This is the simplest test for fetal surveillance, is least expensive and least invasive of all tests for fetal surveillance.
 
Technique
Ref: John Hopkins manual of obs and gynae 4/e, p 90
Cardiff Technique
Sadovsky Technique
Involves maternal counting of fetal movements
When she gets up in the morning and recording the time required for the fetus to move ten times.
On an average this should take 2 to 3 hrs, longer Time periods should prompt the mother to call her Physician for further fetal testing.
Here mother counts fetal movements for 1 hour, this test is usually performed after Meals when the patient lies in left lateral position. To be considered as reassuring 4 or more movements should be present
In an hour. If 4 movements are not felt in First hour, she should repeat it for another
Hour and if 4 movements are not present in 2
Hours, patient should consult her gynaecologist.
 
Management after Abnormal Results
After the observation of decreased fetal movement, the next sequential step to evaluate fetal well being is a non stress test.
60. The answer is (d) i.e She should continue to monitor kick count and return to you after a week to reassess the situa­t­ion
Ref. Williams obs 23/e, p 832 and 839
  • This patient is presenting at 41 weeks with uncomplicated pregnancy and good fetal kick count, i.e there is no fetal distress. I n such a 104situation i.e in uncomplicated pregnancies maximum we can wait uptil 42 weeks for spontaneous labor pains.
  • The international definition of prolonged /postterm pregnancy, endorsed by ACOG is 42 completed weeks or 294 days days or more from the Last menstrual period. Pregnancies between 41 weeks and 1 day and 41 weeks and 6 days, although in the 42nd week, do not complete 42 weeks until the seventh day has elapsed.
  • Protocol for management of pregnancy at or beyond 41 weeks.
Ref. Williams 23/e, p 839
zoom view
Thus it is clear in patients with no complications we can wait till completed 42 weeks of pregnancy.
Since in this patient, cervix is 50% effaced and 1 cm dilated and vertex is at -1 stn. Clinically when I would have got this parient I would have done sweeping and stretching of membranes, and sent her back home and she would have returned after a day or two with labor pains.
61. The answer is (d) i.e LSCS
Again in this question patient is a primi patient who has been dilated to 9 cms for the past 3 hours, i.e it is a case of arrest of dilatation.
 
Remember
Abnormalities of active stage of labor
Protracted Active Phase:
  • Nulliparous–cervix dilates by < 1.2 cms/hr or descent of fetal head is < 1cm/hr
    105
  • Multiparous – cervix dilates by < 1.5 cm/hr or descent of fetal head is < 2cm/hr
  • Arrest of dilatation–no dilatation for >2hrs
  • Arrest of descent–no descent for > 1hr.
In this patient fetal scalp ph was 7.27 earlier and now after 20 mins it is 7.20 signifying fetal acidosis which should be managed immediately by doing a cesarean.
Fetal scalp ph-Fetal capillary scalp blood pH can be used to identify fetuses in serious distress
Ph of scalp = ph of umblical arterial blood and is lower than umblical venous blood
62. The answer is (c) i.e perform immediate cesarean section
 
The Points Worth in this Question
Patient at 40 weeks’ gestation presents in labor, everything is normal, contractions/patients vitals and FHS, cervix is 4 cms dilated and station of fetal head is–3 position i.e it is not engaged, and the moment membranes are ruptured, fetal bradycardia occurs (fetal distress occurs) signifies cord prolapse since the head was not fixed.
Management of Cord Prolapse if Baby is Alive:
  • Emergency cesarean section
  • Forceps delivery only if station of fetal head is below or at +2 station and cervix is fully dilated.
63. The answer is (c) i.e fetal weight
Cord prolapse is an obstetric emergency. After cord prolapse when the cord gets expoed to external environment the whole cord goes into spasm, leading to severe decelerations in fetal heart rate and fetal distress. If the fetus is alive and mature enough for survival, immediate delivery should be done.
Risk factors for cord prolapse include:
  1. Long cord
    106
  2. Polyhydramnios
  3. Abnormal lie (transverse, breech, and oblique)
  4. Multiple pregnancies
  5. Floating (unengaged) head
zoom view
64. The answer is (e) i.e Prolonged second stage of labor due to adequate ute rine contraction strength.
  • Now friends in this question there is a trap, the question which is asked is unrelated to the clinical scenario described before the question.
  • The question is simply asking the condition in which we can use forceps:
    • Option a. Fetal distress during active stage of labor- now friends, active stage of 1st stage of labor begins when cervix is 3 cms dilated and ends when cervix is fully dilated and one of the prerequisites of forceps is that cervix should be fully dilated thus forceps cannot be applied in the active stage of first stage of labor. In case of fetal distress occurring in107
      active stage of labor the only treatment option is cesarean section.
    • Option b. Labor complicated by shoulder dystocia- forceps are not used, management is Mc Roberts manouvre.
    • Option c. Prolonged active stage of labor due to inadequate uterine contraction strength-management is oxytocin to increase uterine contraction strength, again we have already discussed forceps shouldnot be used in active phase of labor.
    • Option d. Prolonged latent stage of labor due to inadequate uterine contraction strength-again latent phase of labor is when cervix begins dilating and lasts uptil it is 3 cms and so forceps cannot be applied.Prolonged latent phase is managed by therapeutic rest.
    • Option e. Prolonged second stage of labor due to adequate uterine contraction strength. Second stage of labor begins when cervix is fully dilated and ends with the delivery of the baby, so logically in this stage forceps can be applied if uterus is contracting (which is so in this patient also)
NOTE: Indications fo r forceps delivery:
  • Fetal distress in the second stage of labor
  • Maternal distress in the second stage of labor
  • Prolonged second stage of labor
  • Prophylactic use of forceps is done when mother has heart disease or PIH, to cut short the second stage of labor.
 
Also Note
In the clinical scenario given before the question patient is a multigravida who is fully dilated (i.e is in second stage of labor) and has been pushing for 3 hours without delivering i.e there is prolonged second stage of labor.
Second stage-normal duration is 0–30 mins for multiparous and 2 hours for nulliparous.
108Prolonged Second Stage is Defined as:
Nulliparous
Multiparous
>2hr
>1hr
Without epidural analgesia
>3hr
>2hr
With epidural analgesia
65. The answer is (a) i.e Fetus should be vertex prestation or face presentation with mentoanterior
66. The answer is (d) i.e occipitomental
67. The answeris (c) i.e low forceps (Ref. Dutta 7/e p 573, 575 williams 23/ed p 513)
F
=
Favourable head position and station
O
=
Open OS (fully dilated)
R
=
Ruptured membranes
C
=
Contractions present and consent taken
E
=
Engaged head, empty bladder
P
=
No major CPD
S
=
Stirrups, lithotomy position
Classification of forceps delivery according to station and rotation
Type of Procedure
Criteria
Forceps used
A High forceps
• Vertex not engaged
• No longer used
Kielland forceps
B Mid forceps
• Head is engaged but presenting part/station is above +2
Andersons or simpsons forceps
C Low forceps
• Station is more than + 2 but has not yet reached the pelvic floor Rotation can be more or less than 45°
D Outlet forceps
• Station is more than + 2 and fetal skull has reached the level of pelvic floor
• Scalp is visible at the introitus without separating the labia
• Fetal head is at or on the perineum
• Sagittal suture is indirect AP diameter or Rt or Lt occipito anterior or posterior position
• Rotation is < 45°
Wrigley's forceps
109
NOTE:
  • The most important point of reference in the use of forceps is the station of Biparietal diameter.
  • Usually forceps should not be applied if head is above + 2 station and if head is unengaged.
 
Presentations in which Forceps
Can be Applied
Cannot be Applied
• Vertex
• Deep Transverse arrest
(only after manual rotation)
• After coming head of breech
(Forceps used-Pipers forceps)
• Face-mento anterior presentation
(Note-Forceps cannot be applied only in those conditions in which cesarean section has to be done)
Transverse lie
• Brow presentation
• Face-mento posterior presentation
 
Coming to Q 65
  • Option a: Fetus should be in vertex or mentoanterior position is correct
  • Option b: Sagittal suture should be less than 15° from antero posterior plane is incorrect
  • Forceps can be applied till maximum 45° rotation
  • Option c: There should be no caput succedaneum- Again incorrect as presence of caput succedaneum is not a contraindication for forceps application
  • Option d: Head should be at zero station also incorrect as outlet forceps is applied when head reaches the perineum
66. Forceps should be applied along occipeto mental diameter of fetus.
67. Extraction of fetus from +2 station by forceps classifies it as low forceps
68. The answer is (b) i.e Face presentation
Attitude of the fetus means the relationship between the different parts of fetus:
110
  • Normally all the parts of fetal body are flexed, i.e the fetus lies in an attitude of flexion, now this is common sense that when there is face presentation the head has to be extended, i.e a deviation from the the normal attitude.
  • Important Terms Frequently Asked
  • Fetal lie: Refers to relation of long Axis of fetus to long axis of mother
zoom view
Fetal presentation- Refers to the part of the fetus which occupies the lower part of the uterus.
Lie
Presentation
Longitudnal
m/c- Cephalic followed by Breech
Thus M/C malpresentation is breech
Transverse
Shoulder
So whether in question it mentions fetus is in Transverse lie or whether they say Shoulder presentation it means the same and should be managed by cesarean. Also in dead fetus with transverse lie best management is cesarean section.
  • Fetal presenting part-It is that part of the presentation which overlies the internal os.
  • In cephalic presentation the most common presenting part is Vertex, since head of the fetus is flexed mostly.
  • Others could be Brow or face presentation, depending on the degree of extention.
NOTE: In brow–always cesarean section is done whereas in face presentation if it is mentoanterior, vaginal delivery can be tried but if it is mentoposterior cesarean has to be done.
  • Fetal position is the relationship of point of direction of presenting part to one of the four quadrants of maternal pelvis. Most common being Left Occipito transverse followed by Left occipito anterior.
  • M/C fetal malposition is occipito posterior position (management is wait and watch).
11169. The answer is (b) Send the patient to labor and delivery immediately for an emergent CS
Now here patient has come for routine antenatal visit, during which it was discovered that the fetus is breech.
Points worth noting are:
  • Gestational age is 37 weeks.
  • On vaginal examination the cervix is 50% effaced and 1- 2 cm dilated.
  • The presenting breech is high out of pelvis, i.e it is not engaged and version can be attempted.
  • The estimated fetal wt. is about 7 lb (i.e it is not much that vaginal breech delivery cannot be attempted)
  • Usg confirms fetus is in frank breech position, there is a normal amount of amniotic fluid present, and the head is well flexed.(all factors favoring version and vaginal breech delivery).
  • Since patient is 37 weeks pregnant and presenting part is high out of the pelvis, amount of amniotic fluid is adequate so we can attempt external cephalic version and yes since version carries a risk of fetal distress and cesarean section. I will give the patient a chance to discuss with her family members and come back to me in a few days if she wishes for external cephalic version. ( i.e option d is correct).
Another option in this case would be to allow the patient to undergo a vaginal breech delivery whenever she goes into labor. i.e option is correct.
Now both version and vaginal breech delivery carry a risk to the fetus so if patient refuses to take any risk I will advise her to go for an elective cesarean section at or after 39 weeks, there is no need /indication for an emergency cesarean section and hence option b is the answer.
70. The answer Version 1 is (b) i:e 24 hrs after delivery
70. The answer Version 2 is (d) i:e during third trimester of pregnancy
(Ref. Internet search)
Kiegels exercise are pelvic floor exercises which consists of contracting and relaxing the muscles that form part of the pelvic floor.
The aim of Kegel exercises is to improve muscle tone by strengthening the pubococcygeus muscles of the pelvic floor.112
Kegel exercises are good for treating vaginal prolapse, preventing uterine prolapse and to aid with child birth in females and for treating prostate pain and swelling resulting from benign prostatic hyperplasia (BPH) and prostatitis in males. These exercises reduce premature ejaculatory occurrences in men as well as increase the size and intensity of erections.
Kegel exercises may be beneficial in treating urinary incontinence in both men and women( The treatment effect might be greater in middle aged women in their 40s and 50s with stress urinary incontinence alone…”.)
Kegels exercises Time for initiating kegels exercise:
  • Pregnancy-1st trimester
  • After vaginal delivery-after 24 hrs
  • After cesarean section-after 24 hrs.
Thus in version 1 answer to the question is 24 hrs after delivery and in version 2 answer is 3rd trimester of pregnancy
Limitations of Kegels Exercises (Ref: Jeffcoates 7/e, p 286)
  • Kiegels exercise has a limited effect as it affects mainly voluntary muscles viz bulbocavernous, levator ani, superficial and deep transverse perineal muscles and not the main fascial supporting tissues.
71. The answer is (a) i.e a. continue breast feeding from both the breasts
72. The answer is (b) i.e dicloxacillin
In q 71: The patient at post partum day 10 is presenting with pain in her right breast.
Her right nipple has become dry and cracked, and it has become increasingly swollen and painful. Her temperature is 38.3°C (101°F). Her right nipple and areola are warm, swollen, red, and tender. There is no fluctuance or induration, and no pus can be expressed from the nipple – these findings are suggestive of mastitis.
In mastitis the recommended therapy is pencillinase resistant beta lactam antibiotics like dicloxacillin and if patient is sensitive to penicillin Erthromycin is the drug of choice.
Mastitis is not a contraindication for breast feeding and patient should continue breast feeding even from the affected breast.
113Similarly in Q72 patient is having mastitis for which the DOC is dicloxacillin.
 
Also know–C/I of Breast Feeding
Maternal Conditions
Fetal Conditions
• Human T cell leukemia
• Undergoing treatment for breast cancer
• Taking any anticancer/teratogenic drug
• Active TB infection
• Active herpes simplex leisons on breast
• Clinically infectious varicella leison
• Galactosemia
• Lactose intolerance
 
Breast Feeding is not C/I in
zoom view
73. The answer is (d) i.e. Warfarin is not a contraindication for lactation
Remember:
  • Warfarin is contraindicated in the first trimester of pregnancy as it can lead to contradi syndrome comprising of microcephaly, optic atrophy, nasal hypolplasia and chondrodysplasia punctate. It can also lead to IUGR, abortions and IUD.
  • But warfarin is absolutely safe during lactation as an extremely minute quantity of it is excreted in breast milk.
74. The answer is (c) ie. Reassurance.
This patient is a puerperal female who is complaining of bloody vaginal discharge with no other signs. O/E there is a sweetish odour bloody 114discharge on the vaginal walls and introitus. Her vitals are normal suggesting that this cannot be PPH (The most common cause of secondary PPH is retained bits of placenta for which curettage is done, but here it is not required).
Slight amount of bloody discharge called as lochia is absolutely normal for the first 15 days after delivery and doesnot require any treatment, so we will reassure the patient and do nothing.
Donot get confused with the finding of WBC count, 10,000 with predominant granulocytes as this is a normal finding in the puerperal period Note- leucocytes can rise to as high as 25ooo during puerperium probably as a response to the stress of labor). Since lochia has no foul smell it means no infection and so no need for culture or antibiotics.
75. The answer is (b) i. e PGF2A
This is a case of Atonic PPH as the patient has presented with vaginal bleeding immediately after delivery and uterus is not palpable per abdominally, i.e it has lost its tone.
In Atonic PPH the first step in management should be uterotonic agents like oxytocin or methylergometrine and if bleeding is not controlled by using either of it, straightaway PGF2a i.e carboprost should be used.
NOTE:
  • If patient would have been an asthamatic then our answer would have been misoprostol since PGF2a is contraindicated in them.
  • If patient would have presented 24 hours after delivery then the answer would have been Dilatation and curettage as the most common cause of secondary PPH is retained placental bits.
76. The answer is (d) i.e factor 11
Pregnancy is a hypercoaguable state, all clotting factors increase in pregnancy except factor 11 and 13.
Another frequently asked question is what happens to fibrinogen levels during pregnancy-since fibrinogen is clotting factor number 1 therefore it also increase in pregnancy.
 
Also Remember
Pregnancy Causes Increase in Concentrations of: (PGI 2001)
  • Globulin
  • Fibrinogen
    115
  • Transferrin and Total iron binding capapcity
  • Leukotrienes.
77. The answer is (c) i.e 36 weeks
 
Anticoagulants in Pregnancy
2 main anticoagulants are:
Warfarin
Heparin
Can cross placenta
Leads to-short stature,
Stippled epiphysis, warfarin and pregnancy
Nasal hypoplasia, pregnancy
Saddle nose and frontal
Cannot cross placenta and so does not lead to fetal defects
But it is not as effective as is a hypercoaguble state.
During pregnancy unfractionated heparin is used.
Bossing if used in 1st Trimester.
Advantage- it is highly effective anticoagulant.
LMWH can be used during pregnancy but should not be used in pregnant patients with valves replaced.
Keeping these things in mind, during pregnancy anticoagulants are used.
Period of gestation
Anticoagulant used
Uptil 12 weeks
12-36 weeks
36 weeks onwards anduptil
6hrs before delivery
From 6 hrs after vaginal delivery and 24 hrs after cesarean
Unfractionated heparin
Warfarin
IV heparin (since if warfarin is continued after delivery PPH can occur)
Restart heparin
3rd day after delivery
Start warfarin and stop heparin once INR is adjusted between 2-3
78. The answer is (c) i.e Pregnancy specific beta1 glycoprotein
Schwangershaft Protein:
  • It is the other name for pregnancy specific B1 glycoprotein.
  • Produced by trophoblast.
  • Can be detected 18 days after ovulation.
  • Its concentration rises steadily and reaches 200mg/ml at term.
  • Role-measure of placental function for fertility control.
11679. The answer is (d) i.e Increased activity of ureters
80. The answer is (b) decr GFR
 
Changes in Renal System During Pregnancy
zoom view
 
Renal System
  • Size of kidney ↑ ‘s (1 cm longer)
  • Dilatation of kidney resembling hydronephrosis
  • Ureter – Dilatation (Right > Left) → occurs in midpregnancy
  • Ureteral elongation which resembles angulations on X-ray.
  • Bladder – due to ureter dilatation = hyperemia occurs.
Now with this background lets have a look at Q79 –In this question there is no doubt about option a i.e increased GFR and option b i.e. Increased RBF during pregnancy. Hypertrophy of the bladder can also occur due to hyperemia but in pregnancy activity of the ureters doesnot increase as there is ureteric stasis due to the smooth muscle relaxant effect of progesterone.
In PIH- B/P of the patient increases so volume of blood reaching kidney decreases as pressure and volume are inversely related, in other words Renal Blood Flow decreases, so GFR decreases. (Ans 80)
81. The answer is (b) i.e 3 – 8 weeks after conception
 
Fetal Growth Periods
Growth Pd
Seen From
Imp Points
Ovulation Pd
Fertilisation-2 weeks
Embryonic Pd
3 weeks-8 weeks
Most threatogenic period
Fetal Pd.
8 weeks and till delivery
82. The answer is (a) i.e Bimastoid
Friends always whenever you get a question on fetal head diameters keep the following points in mind and your question will be solved:
117
Now coming to the question, which has the smallest amongst the given options:
  1. Bimastoid
  2. Bitemporal
  3. Occito frontal
  4. Submento vertical
Here occipitofrontal and submentovertical ar AP diameters and so obviously are bigger than the other two transverse diameters. Between the remaining two, Bimastoid is smaller. (Miss Tina So pretty)
83. The answer is (c) i.e 3rd degree tear.
 
Classification of Perineal Tears
Degree of perineal tear
Structures involved
1st degree
Only vaginal mucosa and perineal skin involved, but not the underlying fascia and muscle
2nd degree
Ilnvolves the vaginal mucosa, perineal skin and the fascia and muscles of perineal body.
3rd degree
Involves the vaginal mucosa, skin and perineal body and the anal sphincter is also disrupted.
4 th degree
In addition, the rectal mucosa is also involved.
11884. The answer is (b) 11-13 weeks
Nuchal translucency is the maximum thickness of the subcutaneous translucent area between the skin and soft tissue that overlies the fetal spine in sagittal plane.
  • Measured between 11–13 weeks
  • NT-upto 3mm is normal and > 3 mm is marker for Downs syndrome(60% cases)
  • Nuchal fold thickness > 5mm is the most imp sonographic marker of aneuploidy in second trimester
 
Causes of Increased Nuchal Translucency
zoom view
85. The answer is (c) Chorionic villi sampling (Ref: Fernando Arias 3/e p 38-40, Williams Obs 23/e, p 300)
Friends: In Patients with previous H/O Down's syndrome the only confirmatory test, which tells us with 100% reliability of the chances of Down's syndrome in present pregnancy is “Karyotyping”
The sample for karyotyping can be obtained in first trimester by–chorionic villi sampling and in 2nd trimester by amniocentesis.
So: Obviously we will think of marking option ‘c’ i.e. chorionic villi sampling as the correct answer BUT. The Question specifically mentions that patient is 9 weeks pregnant and we all know that if CVS is done before 10 weeks– it can lead to limb reduction defects and oromandibular defects in the fetus/Some people argue CVS is not the correct thing to do, at this stage.
Read for yourself what Williams has to say on this issue.
“Early reports of an association between CVS and limb. Reduction defects and oromandibular limb hypogenesis caused a great deal of concern (Burton, 1992; Firth, 1991, 1994; Hsieh, 1995, and all their colleagues). Subsequently, it was shown that limb-reduction defects were associated with CVS performed earlier in gestation—typically around 7 weeks” (Ref. Williams Obs 23/e, p 300)
119
 
Also Know
Diagnostic Tests for Down Syndrome
In first trimester
Chorionic villous biopsy
In second trimester
Amniocentesis
Overall best test for diagnosis is Amniocentesis
Screening test for down syndrome
zoom view
Test not done for diagnosis of Down syndrome–Amniotic fluid estimation.
 
Triple Test - Includes
  • HCG
  • Estriol
  • Alpha feto protein (Maternal).
 
In Downs Syndrome
HCG ↑ AFP ↓ Estriol ↓
(Remember h = for HCG and h for high)
 
In Edwards Syndrome
HCG ↓ AFP ↓ Estriol ↓
Quadrple Test is done for Downs Syndrome
 
It involves Triple Test Plus Inhibin
  1. Alpha fetoprotein
  2. HCG
  3. Estriol
  4. Inhibin: Note levels of inhibin are increased in Downs. Syndrome. (Remember I = for Inhibin and I for increased)
120
 
USG Markers of Downs Syndrome in 2nd Trimester
Echogenic bowel
ASD/VSD
Duodenal atresia (double bubble sign-30% have down)
Cystic hygroma
Single umbilical artery
Ventriculomegaly
Cong diaphragmatic hernia
Choroid plexus cyst
Exomphalos
Sandal gap
Annular pancreas
5th finger middle phalanx hypoplasia
Short femur and humerus
86. The answer is (b) i.e cleft palate
87. The answer is (a) i.e fetal loss (Ref. Williams obs 23/e, p300, textbook of obs, Sheila balakrishnan p 607)
Chorionic Villus Sampling:
  • Done at 10–12 weeks preferebly
  • Done transcervically: 10–12 weeks
  • Done transabdominally: 10 weeks to term.
  • M/C complication is-Fetal loss
  • If done before 9 weeks it results in oromandibular limb defects
  • Chorionic villi sampling can detect
    1. Chromosomal anomaly of the fetus like downs syndrome
    2. Metabolic diseases like- Gauchers disease, Tay sachs disease, Niemann pick disease, Phenylketonuria
    3. Hemoglobinopathies –sickle cell anemia, Hemophilia
But it cannot detect structural defects like Neural tube defects, cleft lip and cleft palate…ans 86
As far as Q 87 is concerned I know most of you will say answer should be oromandibular limb defect…but read the question very carefully, they are asking M/C complication at 4 weeks, now at 4 weeks the gestational sac is hardly formed …no limbs are formed and all cells are totipotent so here the answer will be fetal loss.
88. The answer is (b) i.e 1000 milli IU/ml
89. The answer is (b) i.e The accuracy of determining gestational age using ultrasound begins to decrease after first.
12190. The answer is (c) i.e. Crown rump length on abdominal or vaginal examination.
 
USG in Pregnancy
Earliest sign of pregnancy on USG is–appearance of Gestational sac.
In case of ectopic pregnancy a pseudogestational sac is visible so. The confirmatory sign /First sign of intrauterine pregnancy is Appearance of yolk sac and not gestational sac.
Structure
TVS
TAS
Gestational sac
4 weeks 5 days
5 weeks
Yolk sac
5 weeks
7 weeks
Fetal cardiac activity
6 weeks
7-8 weeks
  • An intrauterine Gestational Sac should be seen by TVS when maternal serum Bhcg level is 1000-1200 micro IU/ml and by TAS with levels of 3000-6000 micro IU/ml.
  • Gestational sac is eccentric and can be seen at 4weeks and 5 days on TVS
  • Double decidua sign of gestational sac is due to interface between the decidua and the chorion, which appears as 2 distinct layers of wall of the gestational sac and differentiates gestational sac from pseudogestational sac of ectopic pregnancy.
 
USG is Done in Pregnancy
  1. To confirm pregnancy
  2. To confirm the no of fetuses
  3. To detect congenital anomaly-
    • – Earliest congenital anomaly detected by USG is anencephaly
    • – Earliest time to diagnose anencephaly-10 weeks
    • – Anencephaly is best diagnosed at 14 weeks
    • – To detect all congenital anomalies single scan should be performed at 16-20 weeks
    • – Marker for Downs in 1st trimester on USG are –Absent nasal bone and increased nuchal translucency
    • – Banana and Lemon sign are seen on USG in case of -Spina Bifida122
  4. To confirm gestational age:
    Trimester
    USG parameter used
    1st Trimester
    CRL-crown rump length
    2nd Trimester
    HC (head circumference)>BPD (biparietal diameter)
    3rd Trimester
    FL (femur length)>HC, BPD
    Overall best parameter and best time to confirm gestational age is
    CRL and first trimester.
  5. To assess fetal growth–Best parameter-AC (abdominal circumference).
Thus Best USG parameter to detect macrosomia as well as IUGR is –Abdominal circumference.
Coming to Q 89- The question is asking about first trimester USG-
Option a.
A gestational sac can be first seen 2 weeks after LMP -incorrect as it is seen at 4 weeks, 5 days after LMP
Option b.
The accuracy of determining gestational age using ultrasound begins to decrease after first trimester- correct as the best time to determine gestational age is first trimester.
Option c.
Yolk sac is the first sign of pregnancy on USG- Incorrect as the first sign of pregnancy on USG is Gestational sac, first sign of intrauterine pregnancy is yolk sac.
Option d.
USG can be used to determine the sex of the baby yes
USG can determine sex of the baby but not in first trimester
Sex of the baby can be determined confirmedly on USG at 14 weeks.
91. The answer is (d) i.e. Congenital nephrotic syndrome (Ref. Williams 23/e, p 291, Fernando arias 2/e, p38)
Alpha Fetoprotein:
  • It is a glycoproteina synthesied by the fetal yolk saca in the early weeks of gestation and by the gastrointestinal tract and liver later.
  • Concentration AFP is maximum in fetal serum at 13 weeks. Maximum concentration of AFP is seen in fetal serum.
    123
  • AFP passes from the fetus to the amniotic fluid by the way of urine and can be detected in amniotic fluid by amniocentesis (diagnostic test)
  • It passes into the maternal serum by diffusion and can be detected in maternal serum after 12 weeks
  • Maternal screening is done between 15-20 weeks. In maternal serum levels peak at 32 weeks.
 
Cut off Levels of AFP- 2.5 Multiple of Median
AFP is Increased
AFP Decreased
Wrong gestational age
G-gestational trophoblastic tumors, gestational diabetes
NTDs
O-overestimated gestational age, maternal obesity
Abdominal wall defects-
A-abortions
Omphalocele gastrochisis
Multiple pregnancy
T- trisomy 21
Renal anomalies
Fetal bladder tumors
Pilonidal sinus
Other Related Points to be Remembered:
  • Fetal serum contains AFP in a concentration 150 times that of maternal serum.
  • Acetyl cholinesterase levels in amniotic fluid is more specific than AFP in predicting Neural tube defects
 
Amniocentesis
Can detect everything which Chorionic villi sampling can detect and in addition since levels of Alpha fetoprotein can be measured, it can detect Neural tube defects.
Therapeutic Indications for amniocentesis
First Half of Pregnancy:
  • Induction of abortion (Instillation of chemicals e.g; hypertonic saline, urea, prostaglandin)
  • Repeated decompression of the uterus in acute hydramnios.
Second Half of Pregnancy:
  • Decompression of uterus in unresponsive cases of chronic hydramnios
    124
  • To give intrauterine foetal transfusion in severe hemolysis following Rh-isoimmunisation.
    • – Time for performing amniocentesis- 16-20 weeks
    • – A 20–22 gauze spinal needle is used.
    • – Length of needle is 4² (014inches or 10 cm)
    • – Amount of amniotic fluid to be collected for diagnostic purposes is 10 ml.
Precautions for Amniocentesis:
  • Prior USG localization of placenta to prevent feto-maternal bleeding.
  • Prophylactic administration of 100 microgram of anti-D Ig in Rh-negative non- immunised mother.
 
Preferred Sites for Amniocentesis
Early Months: 1/3rd of the way up the uterus from symphysis pubis
Later Months:
  • Trans-isthmic suprapubic approach after lifting the presenting part OR
  • Through the flanks in between the foetal limbs OR
  • Below the umbilicus behind the neck of the fetus.
 
Hazards of Amniocentesis
Maternal Complications
Fetal Complications
• Infection
Trauma
• Hemorrhage(placental or uterine injury)
Feto-maternal Hemorrhage
• Premature rupture of the membranes
• Premature labor
92. The answer is (c) i.e. An indicator of considerably increased incidence of major malformations of the fetus (Ref: Williams obs 23/e, p 582)
Single Umblical Artery:
  • Seen in 0.7-0.8% single pregnancy and 5% twin pregnancy
  • Increased incidence is seen in Diabetic pts, black pts, with eclampsia, APH, hydramnios and oligohydramnios
    125
  • Finding of a single umblical is not insignificant as it means increased incidence of congenital malformations(renal/genital), Trisomy 18 and abortion, IUGR, preterm labor in the fetus.
93. The answer is d i.e misoprostol Ref Internet search
Mobius Syndrome:
  • Möbius syndrome is an extremely rare congenital neurological disorder which is characterized by facial paralysis and the inability to move the eyes from side to side.
  • Most people with Möbius syndrome are born with complete facial paralysis and cannot close their eyes or form facial expression.
  • They have normal intelligence
  • Möbius syndrome results from the underdevelopment of the VI and VII cranial nerves
  • Causes: The causes of Möbius syndrome are poorly understood. Möbius syndrome is thought to result from a vascular disruption (temporary loss of bloodflow) in the brain during prenatal development. There could be many reasons for the vascular disruption leading to Möbius syndrome. The use of the drugs misoprostol or thalidomide by women during pregnancy can lead to mobius syndrome.
94. The answer is (e) i.e. ethyl alcohol (Ref: Williams obs 23/e, p)
Maternal abuse-and abnormalities associated.
Smoking Leads most commonly to -IUGR:
  • There are increased chances of preterm delivery, placenta previa, abruptio and Abortion.
  • It leads to Congenital heart defects, gastrochisis and small intestine atresia, cleft lip and palate in the fetus.
Maternal Alcohol abuse-leads to (Ref. Wi lliams Obs 23/e, p317)
Fetal Alcohol Syndrome diagnostic criteria- all required
  1. Dysmorphic facial features-
    1. Small palpebral fissures
    2. Thin vermilion border
    3. Smooth philtrum
  2. Prenatal/and or postnatal growth impairment
    126
  3. CNS abno rmalities:
    1. Structural: Head size < 10 percentile, significant brain abnormalit y on imaging
    2. Neurological
    3. Functional Global cognitive or intellectual deficits, functional deficits in atleast three domains.
Alcohol Related Birth Defects:
  1. Cardiac-atrial or ventricular septal defect, aberrant great vessels, conotruncal heart defects.
  2. Skeletal-radioulnar synostosis, vertebral segmentation defects, joint contractures, scoliosis.
  3. Renal-aplastic or hypoplastic kidneys, dysplastic kidneys, horse shoe shaped kidney, ureteral dilatation.
  4. Eyes-Striabismus, ptosis, retinal vascular abnormalities, optic nerve hypoplasia.
    Ears-Conductive or neuriosensory hearing loss
    Minor-Hypoplastic nails, clinodactyly, pectus carinatum or excavatum, camptodactyly, hockey stick palmar crease, railroad track ears.
Maternal Opiate abuse: After birth children generally appear normal or have small head size,have tremors, irritabilty, sneezing, and vomiting.
  • Symptoms lastfor < 10 days
  • Abnormal respiratory function during sleep can lead to sudden death.
95. The answer is (a) i.e. Glucose challenge test (Ref. Read below)
Points worthnoting here are:
Patient is presenting to antenatal clinic at 24 weeks for routine check up and a coincidental finding on USG is fetus at 24 weeks of gestation in frank breech position, with no other abnormalities.
Now friends at 24 weeks, breech should not worry you as most of the times it spontaneously rotates and be comes cephalic at term. Thus 127options c i.e ECV, d i.e immediate LSCS and e i.e immediate induction and vaginal delivery are ruled out.
Culture for Neisseria gonorhhea and Chlamydia trachomatis. It is normally done at initial visit and in certain high risk groups at 32-36 weeks along with group B streptococcal screening, so it is also ruled out.
24 weeks gestational age is the correct time for screening for gestational diabetes therefore we will do Glucose challenge test with 50 gms of glucose.
Friends, mentioning about breech presentation was just given to confuse you, actually examiner wants to know whether you know the correct time for different screening tests or not.
96. The answer is (d) i.e. FHR ranging from 110-120 bpm over a period of 40 mins.
97. The answer is (c) i.e sinusoidal heart rate pattern.
98. The answer is (e) i.e. Instruct the patient to go to labor and delivery for a nonstress test.
99. The answer is (c) i.e. The results are normal and she can go home (Ref. Management of high risk pregnancy Manju Trivedi, SS puri p 62)
 
CTG IMP Points
Character
Normal
Abnormal
Baseline FHR
110-150 beats per minute
<110 to >150 bpm or variabilty, 5 bpm or presence of decelerations
Baseline
10-25 beats per
<5 bpm for >90 min or
variability
minute
sinusoidal pattern
Accelerations
2 (of atleast 15 bpm lasting for 15 sec) in 20 min
None in 40 min
Decelerations
None
Decrease in FHR by 15 bpm
128
 
Non Stress Test
zoom view
Remember: Whenever a pregnant female complains of decreased fetal movements
  • NST testing should be started 32-34 weeks
  • Interval between NST testing
    • – Ideally – repeated weekly (a reactive NST means that fetus is not in danger for atleast 7 days
    • – In high risk pregnancies like diabetes mellitus, IUGR and Gestational hypertension! twice weekly
    • – In severe preeclampsia remote from term – done daily
  • The positive and negative predictive value of a NST is typically less than 50% and more than 90% which means a reactive NST is more reliable in excluding fetal hypoxia than a non reactive test in predicting fetal compromise
  • A non reactive NST earlier was being followed by a CST (contraction stress test) to confirm the diagnosis of hypoxia but CST is associated with risk of initiating labour and hence these days a non reactive NST is being followed by a biophysical score (Manning score)
129
zoom view
Decelerations-decrease in FHR of 15bpm:
  • Early-(Type1)-Head compression/vagal nerve stimulation
  • Late (Type 11)-uteroplacental insufficiency and fetal hypoxia
  • Variable deceleration (M/C type) -cord compression
  • Sinusoidal heart rate pattern - Stable, fixed heart rate with fixed variabilty and without any accelerations. Seen in fetal anemia and fetomaternal haemorrhage.
 
BPS, Interpretation and Pregnancy and its Management
BPS
Interpretation
Mgt
Fetus normal
High risk-repeat weekly
Diabetic /post term/ non reactive NST- twice weekly
8 normal liquor
Normal
Same
8 liquor less
Chronic fetal asphyxia
If >37 wks, deliver otherwise repeat testing
Possible fetal asphyxia
Amniotic fluid-less- deliver
Normal amniotic fluid->36 wks-deliver
Repeat test-<6-deliver
Repeat test>6 repeat test as per protocol
Probable fetal asphyxia
Repeat test on same day- if <6-deliver
Certain fetal asphxia
Delivers
Now with this background see the Q's.
13096. Hypoxemia/concerning NST would be the one in which FHR is ranging from 110–120 bpm over a period of 40 mins (i.e. twice NST is done no accelerations seen)
97. Fetal anemia leads to sinusoidal heart rate pattern.
98. Patient is complaning of decreased fetal movement, do not take it lightly send her for NST.
99. BPS shows a score of 8 which is normal, so send the patient back home and reassure.
100. The answer is (c) i.e. Taking vitamins before conceiving and continuing them after conceiving
The points worth noting here are- patient is presenting to the prenatalclinic for the first time at 26 weeks of gestation. USG shows herniation of cerebellum through foramen magnum and frontal bossing and lumbosacralkyphosis.
Herniation of cerebellum through foramen magnum is Banana sign and frontal bossing is lemon sign, both of which can be seen in Spina bifida which can be prevented by taking folic acid before conceiving and continuing it after conception.
 
USG Signs of Spina Bifida
The intracranial signs which aid in diagnosis are:
  • Small BPD
  • Venticulomegaly (> 10 mm)
  • Frontal bone scalloping- Lemon sign – as shown in the usg given in the question.
  • Elongation and downward displacement of cerebellum-called as Banana sign(banana sign disappears after 24 weeks)
  • Obliteration of cisterna magna
    NOTE: Ninety nine percent of fetuses with NTD have one of the above-mentioned 5 specific cranial abnormalities.
    Lateral ventricle is measured at the atrium-normal =7mm from15 wks onward
  • Mild-ventriculomegaly-10-15 mm
  • Overt->15 mm
  • A dangling choroid plexus is seen in severe cases
    131
 
TRUE AND FALSE
101. SLE can be managed during pregnancy using prednisolone, sulphsalazine and methotrexate- is false. (Ref Williams 23/e, p 1150)
Methotrexate cannot be used during pregnancy—“Medicine to be avoided in lupus include mycophenolate mofetil and methotrexate”.
(Ref Williams 23/e, p 1150)
 
Drugs Used in Management SLE
Arthralgia
Severe disease
Skin rash
• NSAIDS
• Sulphasalazine
• Corticosteroids
• Azathioprine
• Cyclophosphamide- not in life threatening situations
• Chloroquinine
 
Drugs to be Avoided
  • OOCPs and IUCD
  • Mycophenolate
  • Mehotrexate
102. Internal podalic version is done under GA is true. (Ref. Dutta obs 7/e, p 585)
 
Internal Podalic Version
Only indication:
  • Second twin transverse lie
  • Always do under GA
  • M/C complication-Rupture uterus
103. A 31 year old pregnant female with twin gestation has more chances of Downs syndrome than general population is true.
Downs syndrome–Chances of Downs are increased at the following material ages.
  • ≥ 35 yrs   In singleton pregnancy
  • ≥ 31 yrs   In twin pregnancy
  • ≥ 31 yrs   In h/o previous down syndrome
132104. Hayman and Cho square sutures are used for managing PPH-true
Sutures used in PPH mgt.
zoom view
105. Placenta previa mostly needs cesarean section if placental edge lies within 2 cms of the os True Dutta obs 7/e, p249 (Ref. Williams obs 23/e, p773)
According to Williams: Cesarean delivery is necessary in practically all patients with placenta previa.
According to Dutta: Cesarean delivery is done for all women with sonographic evidence of placenta previa where placental edge is within 2 cms from the internal os.
106. PPH has a tendency to recur in subsequent pregnancy… true (Ref. Dutta obs 7/e, p 412)
For preventing PPH it is important to identify certain risk factors antenatally, one of which is previous history of PPH. So PPH has a tendency to recur in future pregnancies.
Ans. 107 Direct coombs test detects maternal Ig M on fetal red cells… false (Ref. Dutta obs7/e, p 335)
Indirect and Direct coombs test Ig G antibody and not Ig M as Ig G can cross the placenta and not Ig M.
108. Listeria infection may be suspected if meconium is present in amniotic fluid in < 34 weeks gestation…True (Ref. Williams obs 23/e, p 1224)
“Discoloured, brownish, or meconium stained amniotic fluid is common with fetal infection, even with preterm labors.”
Ref. Williams obs 23/e, p 1224
109. DOC for pneumocystis carinii in pregnancy is pentamidine…False (Ref. Williams obs 23/e, p 1252)
According to Williams and internet search DOC for treatment of pneumocystis carinii during pregnancy is sulfamethoxazole-trimethoprim.
133110. Peak systolic velocity is increases in middle cerebral artery with fetal anemia.. True
‘Middle cerebral artery peak systolic velocity It is a noninvasive tool for the diagnosis of fetal anemia. The principle behind this test is that there is increased velocity of blood flow in the anemic fetuses due to increased cardiac output in an attempt to enhance the oxygenation.”…. High Risk Pregnancy SS trivedi, Manju Puri p 265.
 
DOPPLER STUDY- Important points
 
Indications of Doppler study during pregnancy
  • IUGR (most important investigation for management)
  • Rh iso immunization
  • Prediction of PIH
  • Diagnosis of placenta accreta/ percreta, vasa previa.
 
Uterine Artery: Doppler
Increased impedance of maternal uterine artery velocimetry at 16-20 weeks is predictive of preeclampsia and IUGR.
 
Middle cerebral artery (MCA)
In fetus with IUGR, as the S/D ratio begins to rise the blood flow in MCA increases. There is redistribution of blood flow and vital organs like brain continue to receive adequate blood at the expense of liver and kidney. This is called brain-sparing effect.
Peak systolic velocity (PSV) in the middle cerebral artery is increased in fetal anemia. PSV in MCA is now used in management of Rh isoimmunized fetuses.
Umbilical artery:
  • – A normal systolic/ diastolic (S/D) ratio indicates that the fetus is receiving adequate blood supply.
  • – Rising S/D is the earliest change in IUGR.
  • – Absence of diastolic flow in umbilical artery is an ominous sign and Intrauterine fetal death can be expected within 7 days.
    134
  • – In extreme cases diastolic flow may become reversed and IUFD will occur with in 48 hours.
111. A 30 year old female with BMI 28kg/m2 should gain 14-16 kg weight during pregnancy….False (Ref. Williams obs 23/e, p 950 table 43-3)
Weight gain during pregnancy
Category
BMI(kg/m2)
Recommended weight gain(kg)
Pounds
LOW
<18.5
12.5 TO 18
28-40
NORMAL
18.5-25
11.5-16
25-35
HIGH
26-29.9
7-11.5
15-25
OBESE
>30
5-9.1
11-20
Factors affecting weight gain:
zoom view
NOTE: Cigarette smoking doesnot affect maternal weight gain, it affects fetal weight gain.
112. Montevideo unit is contraction in mm of H2O per 10 mins…false (Ref. Williams obs 23/e p437)
 
Montevideo unit
Montevideo unit is strength of contraction in mm of hg multiplied by frequency per 10 min, for eg 3 contractions in 10 min each of 50mm of hg = 150 MVU
Clinical labor begins at 80-120 MVU
During active stage uterine contractions are 190-300MVU
113. M/c cause for second trimester recurrent abortions is cervical incompetence …True (Dutta obs 7/e, p 168-169)
135
zoom view
1st trimester abortion-M/C cause
2nd trimester abortion-M/C cause
• Isolated-chromosomal anomaly-trisomy16
• Isolated-cervical incompetence
• Recurrent-chromosomal anomaly-balanced translocation of chromosome
• Recurrent-cervical incompetence
• TORCH infections never lead to recurrent abortions
2 most useful investigations for recurrent abortions-
• Karyotyping
• Testing for antiphospholipid
Ab
Most common cause of recurrent abortions-
• 1st trimester- chromosomal defect
• 2nd trimester-Cervical incompetence
 
Cervical Incompetence
• Diagnosis in non pregnant state-H/O painless spontneous abortion in 2nd trimester
• Hegar dilator no 8 can be passed without resistance
• HCG-funnel shape appearance
• Management
• Vaginal circlage/ Abdominal circlage
• Time for stitch application -after 14 weeks
• Time for stitch removal- 37 weeks or earlier in preterm labor
• Diagnosis in pregnancy-
• TVS-Cx length <30 mm and diameter >20 mm
• C/I of stitch application-
• Cx> 4cms dilated
• Memb ruptured/ vaginal bleeding/ uterusirri- table/ infection present
114. Trial of labour is C/I in previous LSCS pts….True
Now friends there is a hell lot of difference between Trial of labor and VBAC
136TRIAL OF LABOR-Trying to Conduct spontaneous vaginal labor in mild to moderate degree of inlet contraction of the pelvis i.e obstetrician is aware pelvic inlet is mildly contracted and still vaginal delivery is being tried.
VBAC-Vaginal birth after cesarean is –when vaginal delivery is being tried in a previous LSCS patient, only if pelvis is normal and not contracted.
 
C/I OF TRIAL OF LABOR
  1. Associated midpelvic or outlet contraction
  2. Presence of complicating factors like-
    • – Elderly primi
    • – Malpresentation
    • – Post maturity
    • – Post cesarean section
    • – Preeclampsia
    • – Medical disporders like DM, heart dis.
This simply means that in a previous LSCS patient if pelvis is even mildly contacted donot try for vaginal delivery, just go for repeat cesarean.
 
Also know
  • In Contracted pelvis-obstetric conjugate <10 cms
  • If Obstetric Conjugate is 9.6 cms to 10 cms –mild contraction
  • If it is 8.6 to 9.5 cms –moderate contraction
  • If <8.5 –Severe contraction
115. Pelvis with AP diameter more than transverse diameter is Android pelvis…False (Ref. Dutta obs 7/e,p 346 table)
Types of pelvis-Caldwell and Mohoy classification
Gynaecoid
Android
Anthropoid
Platypelloid
M/C type
Male pelvis
Ape like pelvis
Flat pelvis
Least common
Inlet transverse diam>AP diam
Shape of in let-round/ transverse oval
Inlet-transverse diam>AP diam
Shape of inlet- wedge shaped
Only pelvis in which
AP sdiameter >transverse diameter
Shape-long oval
Bowl shaped pelvis Trans diameter much more than AP diameter
137
Subpubic
angle-90
Subpubic
angle-<90
M/C position – Left occipito
Transverse is the most common followed by Left occipito anterior
Deep Transverse
Arrest/Non rotation
Direct occipito posterior position common
Engagement- synclitic/ asynclitic
Naegles-ant parietal bone leads, seen in multi Litzman- post parietal bone leads, seen in primi
Dystocia dystrophica syndrome
Face to pubes del can occur
Engagement occurs by exagerrated asynclitism so engaging diameter is super subparietal diameter.
116. Loss of knee jerks occurs when Mg SO4 conc->8 meq…False (Ref. Dutta 7/e, p 234, Fernando arias 3/e, p420–421, Williams ob s 23/e, p738–739)
 
Mg SO4
  • DOC to prevent and treat convulsions in eclampsia
  • Mech-decr acetylcholine release and blocks ca channel(so use carefully with Ca channel blockers)
  • Therapeutic range-4 to 7 meq/l
  • First sign of toxicity- loss of knee jerks->10 meq/l
  • Respiratory depr occurs if mg conc >12meq/l
  • Regime followed-Pritchard regime
117. A 25 year old primigravida with 24 weks gestation has a first episode of asymptomatic bacteriuria. The risk of having pyelonephri tis is 15%….False (Ref. Williams 23/e, p1035–1036, COGDT, 10/e, 374)
138Asymptomatic Bacteriuria:
  • Bacterial Count >105 /ml in midstream clean catch sample on 2 occasions in an asymptomatic patient is referred to as Asymptomatic bacteriuria.
  • M/C organism -Ecoli
  • Incidence in pregnancy and non pregnant -1 to10%
  • m/c in multiparity, diabetic patients and sickle cell trait patients.
Screening test-Dip slide test
ACOG-recommends screening at first prenatal visit
Risk of pyelonephritis in untreated-25%, in treated-10%
Risk of UTI-40%
T/t-ampicillin,amoxicillin,cephalexin or nitrofurantoin.